You are on page 1of 162

2017 11.

Movement of material across the cell membrane


ENTRANCE EXAMINATION TO SCHOOL which does not requiring expenditure of metabolic
OF NURSING UNIVERSITY OF BENIN energy is called (a)Active transport
TEACHING HOSPITAL, USELU, BENIN (b) Co-transport (c)Counter transport
CITY GENERAL NURSING PAST (d)Passive transport (e)Retrograde transport
QUESTIONS AND ANSWERS 12. A group of ribosomes attached to mRNA is known
as
OPTION C (a) Polymer (b)Polypetide (c)Polysomes
1. Which of the following metals cannot replace (d)Manomer (e)Monosomes
hydrogen from water or steam? 13. Example of ovavivparous
(a) Sodium (b) Magnesium (c) Iron (a)Reptiles (b)Mammals
(d) Calcium (e) Copper. (c) Duckbill platypus (d)Frog (e)Birds
2. The most common type of chemical reaction
which alkanes undergone is 14. During birth which of the following act as birth
(a) Substitution (b) Addition canal? (a)Oviduct
(c) Condensation (d) Polymerization (b)Fallopian tube (c)Uterus (d)Virgina
(e) Double decomposition. (e)Vas deferens
3. By means of filtration, one component can be 15. Nodules on root of leguminous plants have
obtained pure from an aqueous mixture of sodium Bacteria (a)Anarobic
chloride and (b)Nitrogen fixing (c) Nitrifying
(a) Potassium nitrate (b) Sand (d) Ammonifying (e) Saponifying
(c) Lead nitrat (d) Glucose (e) Starch 16. Parasites capable of living independently of its
4. Which of the following substances are all made by host at times are called parasites
the process polymerization? (a)Obligate
(a) Nylon and soap (b) Strict
(b) Ethanoic acid, margarine and ethanol (c)Facultative
(c) Nylon and artificial rubber (d)Endoparasite
(d) Soap and butane (e)Free Living
(e) Margarines and nylon 17. Hepatic and pancreatic secretions are also
5. The oxidation state of manganese in potassium stimulated by a hormone produced by the
permanganate is intestinal mucosa. The hormone is
(a) +7 (b) +5 (c) +3 (d) +2 (e) +1 (a) Creatinv (b)Gastrin (c)Secretin
6. The empirical formula of an oxide of nitrogen (d)Pepsin (e)Lipase
containing 30.4% of nitrogen is (N= 14.0, O=16.0) 18. DNA contains
(a)N2O2 (b)NO (c)NO2 (d)N2O (e)N2O2 (a) Purines (A and G) pyrimidines
7. Chlorination of water for town supply is carried (U and C)
out to: (b) purines (T and (c) pyrimidines (A and G)
(a) Make the water colourless (c) purines (A and (c) pyrimidines (U and G)
(b) Remove germs from the water (d) purines (A and G) pyrimidines (T and
(c) Make the water tasteful (c)
(d) Remove odour from the water (e) MRNA and TRNA
(e) Make the water tasteless 19. Which one is a micromolecule?
8. Which of the following metals is least reactive? (a)Polysaccharide (b)Protein
(a)Pb (b)Sn (c)Hg (d)Au (e)Na (c) Hemoglobin (d)ATP (e) Myoglobin
9. The purest form of iron which contains only 0.1% 20. Chlorophyll molecule contains as central
carbon is : (a)Pig iron (d)Alkanamide (e)Alkanostic
2 2 acid 2 2
(b)Wrought Iron
(c)Cast iron
(d) Iron pyrite
(e)Strong iron

10. The final oxidation of alkanol, alkanal and


alkanones is: (a)Alkanoic acid
(b)Alkanoyyl halide (c)Alkanomide
metal on (a)Fe + (b)Mg + (c)Zn + (d)Cu +
(e)Al +
21. Nigeria‘s newest state to be
categorized as Oil producing state is
(a) Anambra
(b)Enugu (c)Lagos
(d)Borno (e)Rivers
22. The following state in Nigeria are
named after rivers in their territories
except (a)Kaduna (b)Rivers (c)Imo
(d) Sokoto
(e) Cross River
23. UNIBEN Teaching Hospital is
located in Local Government Area
(a) Egro the objectives of the association
(b)Ikpoba (a) Harmful
(c)Ovia South West
(d)Ovia North East
(e)Oredo
24. The capital of Lagos State is (a)Lagos
(b)Victoria Island (c)Lekkipenusila
(d)Surulere
(e)Ikeja
25. The Secretary General of the United Nations is

(a) Ban Ki Moon


(b)Largede
(c)Boutrus Galli
(d)Emeka Onyeaku
(e)Serena Williams
26. Nigeria‘s Minister of petroleum is
(a) Alison Madueke (b)Ibekachukwn
(c)Muhammadu Buhari
(d)Tam David West (e)B Lawal
27. Africa‘s youngest independent State of Nigeria
is (a)Sudan
(b)Eritrea (c)South Sudan (d)Guinea Bisau
(e)South Sudan
28. Sambisa Forest is located in which State of
Nigeria
(a) Yobe (b)Borno (c)Adamawa
(d)Taraba (e)Gombe
29. The honourable Minister of Health of Nigeria is
(a)Onyekachi Chukwu
(b)Khaliru Alhassan (c)Babatunda Osotimehin
(d)F Adewole (e)Osagle Ehanire
30. The capital of Economic Community of West
Africa States (ECOWAS) is located in
(a) Addis Ababa (b) Naiaobi (c) Harare
(d) Abuja (e) Dar essalam

From the words or group of words lettered A-E


below each of the following sentences, choose the
word or group of words that is nearest in meaning to
the underlined group of words as used in the sentence
31. But for the principal actor the play would have
been dull
(a) important (b) head (c) master
(d) famour (e) main
32. He was reluctant to grant my request
(a) disposed (b) Delighted (c) Reticent
(d) Unwilling (e) Agreeable
33. The military government called for a concerted
effort in solving the problems of the state
(a) Dramatic (b) An agitated (c) A Joint
(d) A directed (e) An unfailing
34. The chairman is of the opinion that
accepting the proposal would be inimical to
(b) Relevant
(c) Irrelevant For question 41-45 choose the
option A-E the one that is
(d) Indispensible
(e) Helpful
35. Kunle is very pessimistic about our
chance of success
(a) Sad
(b) Despondent
(c) Unconvinced
(d) Worried
(e) Concerned

From the words lettered A-E, choose the word


that best complete each of the following
sentences
36. Despite all our leas, the manager remained
in his refusal of our request.
(a) Flexible
(b) Pilable
(c) Adamant
(d) Unconvinced
(e) Curious
37. Poor feeding causes in children
(a) Maladjustment (b) Malfunction
(c) Male (d) Malaria (e) Malnutrition
37. Poor feeding causes……… in children
a) Maladjustment
b) Malfunction
c) Malediction
d) Malaria
e) Malnutrition
38. It is a that more you look the less you see
(a) Wonder
(b) Feat
(c) Myth
(d) Paradox
(e) Magic
39. Femi has a Appetite
(a) Vivacious
(b) Loquacious
(c) Garrulous
(d) Suspicious
(e) Voracious
40. His appointment was because of
the irregularities discovered in his
credentials
(a) Cancelled
(b) Terminated
(c) Suspended
(d) Stopped
(e) Postponed
MOST OPPOSITE IN MEANING TO (b) 11.1% (c) 10.1%
the underlined word (d) 9.1%
41. Although people say that Tunde is respectful, I
find him quite
(a) Discourteous
(b) Immature
(c) Inefficient
(d) Loguacious
(e) Respectable
42. He was instructed to catch the thief dead or

(a) Alive
(b) Deceased
(c) Killed
(d) Living
(e) Unconscious
43. Some people are wallowing in abject penury while
others are swimming in ___ _
(a) Affluence (b) Comfort (c) Happiness
(d) Property (e) Success
44. Mr. Ojo is notably a devoted Christian but his wife
is best described as Christian
(a) Controversial (b) Nominal (c) Passive
(d) Regular (e) Regular
45. The principle announced that the cleaning of the
school garden would be voluntary and not _
(a) Differed
(b) General
(c) Necessary
(d) Obligatory
(e) Optional
46. Four pupils have an average age of 12 years and
two other pupils of average age of 10.5 years.
What is the average stage of all the pupils?
(a) 10.5 yrs
(b) 11 yrs
(c) 11.5 yrs
(d) 12 yrs
(e) 12.5 yrs
47. If the mean of the following set of
Data:2,3,3,3,2,4,4 and 3 is divided by the mode,
the result is
(a) 1
(b) 2
(c) 3
(d) 4
(e) 5
48. If a sales boy gave a change of #72.00 instead of
81.00, calculate his percentage error to correct
one decimal place (a) 6.0%
(e) 13.1% and 4 bottles of mirinda cost #380.00
49. The last Monday of the 31 day month, must be altogether, how much does a bottle of malt
at least how many days after the first Friday of
the same month? cost?
(a) 20 (a) #90.00
(b) 24
(c) 21 (b) #80.00
(d) 30
(e) 35
50. What is the total surface area of a cuboid
whose dimensions are 12c x 8cm x 3cm?
(a) 2cm2
(b) 96cm2

(c) 36cm2

(d) 24cm2
(e) 12cm2
51. A machine valued at #20,000 depreciates by
10% every year. What will be the value of the
machine at the end of 4 years? (a) 16,200
(b) 12,000
(c) 18,000
(d) 16,000
(e) 120,000

52. For an arithmetic sequence, the first term


is 2 and the common difference is 3. Find
the sum of the first 11 terms
(a) 157
(b) 168
(c) 197
(d) 200
(e) 187
53. A petrol attendant recorded the number of
vehicles that bought petrol at the station
for two hours and result is 12 cars and 27
buses. Find the probability that the next
vehicles will be a bus
(a)
4/39
(b)
4/13
(c)
5/13
(d)
6/13
(e) 3/5
54. A bottle of malt costs three times as much
as a bottle of mirinda. If 5 bottles of malt
(c) #60.00 (e) Use any of these methods

(d) #40.00
(e) #420.00

55. A variable y is inversely proportional to x.


what is x, when y = 10?
(a) 0.1
(b) 2
(c) 100
(d) 1.0
(e) 2.0
56. A palm fruit dropped to the ground from the
top of a tree 45m tall. How long does it takes
to reach the ground?
(a) 9s
(b) 4.5s
(c) 6s
(d) 7.5s
(e) 3s. (g=10m)
57. The internationally agreed system of units (SI)
for physical measurements are
(a) Ib, ft, sec
(b) g, m, s
(c) kg ,m, sec
(d) cm ,g, sec
(e) kg, cm, sec
58. To determine the weight of an object you
could
(a) Use of beam balance
(b) Use of spring balance
(c) Find the force necessary to give certain
acceleration
(d) Use none of these methods
59. On an A.C generator, which of the following
does not apply?
(a) A field magnet
(b) Commutator
(c) Armature
(d) Slip rings
(e) Carbon brushes

60. An object 1cm high placed on the axis 15cm


from a converging lens forms an image
30cm from the lens. The size of the image is
(a) 0.5cm
(b) 1.5cm
(c) 3cm
(d) 2.5cm
(e) 2cm.
ANSWER TO 2017 NURSING
PAST-QUESTIONS 10. Ans: A
Alkanoic acids have general formula of
SECTION: A CnH2n+1 COOH

1. Ans: E 11. Ans: D


Reason: Copper is below hydrogen in the When transport required energy, it is said to
electrochemical series (ECS). The other listed be ACTIVE.
metals are above hydrogen.
12. Ans: C
2. Ans: A RNA types include: messenger RNA
Alkanes are saturated hydrocarbons with (mRNA): transfer RNA (t RNA): and
general formula CnH2n +2 ribosomal RNA (rRNA)

3. Ans: E 13. Ans: A


Sodium chloride is soluble in water but starch Snakes are a classical example. Ovoviparous
is not upon filtration, pure starch is the residue animals produce eggs but do not lay them.
while pure sodium chloride becomes the Instead the eggs mature within the mother.
filtrate
14. Ans: D
4. Ans: C The oviduct is the fallopian tube. The uterus
Nylon and artificial rubber are polymers. houses the foetus prior to birth. Vas deferens
None of the item in the other options are is a part of the reproductive system.
exclusively polymers.
15. Ans: B
5. Ans: A These nitrogen fixing bacteria belong to the
Potassium permanganate (KMnO4) is also Rhizobuim group
called potassium tetraoxomanganate (VII)
K + Mn + 4 = 0
(=1) + (Mn) + 4(-2) = 0
Mn + 1 – 8 = 0 16. Ans: C
Mn – 7 = 0 Obigate parasite definitely require their host to
Mn = 0 + 7 = +7 complete their life cycle viruses are examples.

6. Element N O 17. Ans: C


!"#$"%&'(" !"#$"%&'&"( !".! !".! Secretin regulates water homeostasis and
!"#$%&!"## !" !"
stimulates secretions from the stomach,
2.17 4.35
divide through by the smaller value pancreas and liver
!.!" !.!"
!.!" !.`" 18. Ans: D
Empirical formula = CH2 Wrought iron is purest while pig iron is most
7. Ans: B impure.
Chlorine is a germicide

8. Ans: D
Au is the symbol of gold (aurum). Gold
occupies the bottom position in the
electrochemical series

9. Ans: B
The purines in DNA and RND are adenine
(A) and guanine (G). the pyrmidines in
DNA are crytosine (C) and thymine (T)
while those in RNA are cytosine (C) and
uracil (U)

19. Ans: D
Micromolecules are small/tiny molecules.
The other listed molecules are
macromolecules.

20. Ans: B
Chlorophyll is a parphyrin of magnesium
(mg2+) whereas haemoglobin is a parphyrin of 31. Ans: E
iron and contains ferrous (Fe2+) iron

21. Ans: C
Lagos state was the 11th state recognized as oil
producing.

22. Ans: D
Sokoto state is named after the Sokoto
caliphate

23. Ans: D
The GENERAL TEACHING teaching
hospital was founded in 1973

24. Ans: E

25. Ans: A
Ban Ki moon was succeeded by Antonio
Guterres from Portugal who assumed offices
on 1st January 2017.

26. Ans: C
President Muhammadu Buhari succeeded
Diezani Alison MAdueke as petroleum
resources Ministry. Madueke was the first
female president of OPEC.

27. Ans: C and E


South Sudan became a sovereign state on July
9th 2011.

28. Ans: B
Sambisa forest served as hideout for the
notorious BOKO HARAM sect.

29. Ans: D
Professor Isaac Folorunso adewole is a
professor of obstetrics and gynecology, former
VC of the University of Ibadan and was
appointed Health Ministry in November 2015.

30. Ans: D
The ECOWAS was established on 28 May
1975. It comprises 15 member states, eight
French-speaking. Five English-speaking and
two-Portuguese speaking.
“Main” means ―most important‖ same as “Alive” means ―living; not dead‖ the opposite
―principal‖ is ―dead‖

32. Ans: D
“Unwilling” means ―not wanting to
do something and refusing to do it‖ same as
―reluctant‖

33. Ans: C
“A joint” means done together by more
persons same as ―concerted‖

34. Ans: A
“Harmful” means ―something that
is destructive or can cause pain‖ same as
―inimical‖

35. Ans: B
“Despondent” means ―sad‖, without much
hope‖
36. Ans: C
“Adamant” which means ―determined not
to change your mind or to be persuaded
about something‖

37. Ans: E
“Malnutrition” means ―a poor condition
of health caused by a lack of food or a lack
of the right type of food‖

38. Ans: D
“Paradox” means ―a statement
containing two opposite ideas that make it
seem impossible or unlikely, although it is
probably true‖

39. Ans: E
“Voracious” means ―eating or wanting
large amount of food same as greedy‖ or
―avid‖

40. Ans: B
“Terminated” means ―to end; to
make something end‖
41. Ans: A
“Discourteous” means having bad manners
and not showing respect for other people.
The opposite is ―respectful‖

42. Ans: A
43. Ans: A S11 = 5.5 4 + (10)3
“Affluence” means ―having a lot of money S11 = 5.5 4 + 30 = 5.5 x 34
and a good standard of living‖ same as S11 = 187
prosperous, wealthy, rich etc. the opposite is 53. None of the options is correct.
―penury‖ Total No of vehicles = 12 + 27 + 39
Pr(bus) = !" !" !"#$# = !" = !
44. Ans: B !"!#$!" !" !"
“Nominal” means being something in name
only and not in reality‖ the opposite ―devoted‖ 54. Ans:C. Let a bottle of melt cost ―x‖ while a
bottle of mirinda cost ―y‖
Then x = 3y – (1)
45. Ans: D 5x + 4y = 380 = (2)
“Obligatory” means that which you must do From (1) above
because of the law or rules etc‖ the opposite is X = 3y
―voluntary‖ or ―optional‖ Substitute 3y for x in (2) above
5(3y) + 4y = 380
15y + 4y = 380
46. Ans: C 19y = 380
Total age of the first four pupils Y = !"# = 20
!"
= 12 X 4 =48years. Total age of last two Substitute 20 for y in (1)
pupils = 10.5 x 2 = 21yrs X – 3(20) = N60
Total age of 6 pupils = 48 + 21 = 69
average age of all pupils = 69/6 = 11.5 yrs 55. None of the options is correct insufficient
details
47. Ans: A Y∝ 1
Give 2,2,3,3,3,3,4,4,
Mode = (appears the most) 3 Y= ; x =y = 10,
When
Mean = !!!!!!!!!!!!!!! = !"
! ! x=
=3
∴ =3 3 56. Ans: E
=1 S = ut + 1 2 gt2
48. Ans: B !""#" U=0(as the fruit falls from rest)
Percentage error = !"#$!% !"#$%x 100 t=?
Error = 81 – 72 = N9.00 g = 10m/s2 (given)
Actual value = N81,00 S = 45m
% error = ! X 100 = 11.1% 45 = 0 x t + 1 2 (10 x t2)
= 5.5 2(2) + (11 – 1)3
49. Ans: B
50. Ans: A. TSA = 2(l x w) + (w x h) + (h x l)
TSA = 2(12 x 8) + (12 x 3) + (8 x 3)
= 296 + 36 + 24 2
= 2156 = 312cm

51. Ans: B. At the end of four years, the total depreciation


would be 40%
40% of N20,000
!"
= x 20,000 = N8,000
!""
New value = N20,000 – N8,000 = N12,000

52. Ans : B
S11= 2 2a + (n – 1)d S11
45 = 1 2 (10 x t2)
45 = 5t2
45 = 5t2
t2 = 45 = 9
t = 9 = 3s

57. Ans: C
Kilogram is the S.I unit of
mass (kg) Metre is the S.I unit
of length (m) Second is the S.I
unit of time (s)

58. Ans: E
Remember the following:
Weight = mass x acceleration due to
gravity Force = mass x acceleration
≠ Weight is also a force and is thus (d) Fertilizer
measured in N.
2. The movement response of a cockroach away
59. Ans: B
from a light source can be described as
The others are parts of AC generator.
Commutations are found in the DC motor 1. A food chain begin with a
!"#.!!"#!! !"#$% !!"#!! ! ! (a) Decomposer
60. !"# !"#$%&'(
= !"#$% !"#$%&'(
= = !" !"
(b) Producer
x = !"! ! = 2cm. (c) Primary consumer
!"

2014
ENTRANCE EXAMINATION TO SCHOOL
OF NURSING UNIVERSITY OF
BENIN TEACHING HOSPITAL,
USELU, BENIN CITY
NURSING PAST QUESTIONS AND
ANSWERS
SATURDAY 26TH JULY,
INTRODUCTION TO CANDIDATES
1. You are to answer the 50 multiple choice type
question in 75 minutes
2. A question is represented by a sentence or
phrase followed by the stems or suggestion
out of which you are to make a correct choice.
3. The choice made should be indicted by
shading the corresponding box with HB
pencil. Any erasure must be thorough.
4. Questions booklets are in four option A-D you
are to ensure that you shade the option that is
on your question booklet unto the appropriate
box on the O?MR form.

5. SUBMIT BOTH QUESTION PAPER AND


OMR FORM UNHAMPERED WITH
AND SPARATELY AT THE END OF
THE UNANSWERED QUESTIONS
6. MARKS WILL BE DEDUCTED FOR
WRONG ASWERS
7. THERE SHALL BE NO PENALTY FOR
UNANSWERD QUESTIONS
8. PHONE, CACULATORS OR FOUR
FIGURES TABLES ARE NOT
ALLOWED INTO THE HALLS.
9. WRITE YOUR NAME BOLDLY ON THE
QUESTION BOOKLET-SURNAME FIRST

Each Candidate must sign the space provided


for signature.

Signature of candidate ……….


(a) Negative phototropism
(b) Positive photropism
(c) Positive phototaxism
(d) Negative phototaxism
(e) None of the above
3. The vector for Yellow fever is ?
(a) Anopheles mosquito
(b) TseTse fly
(c) Black fly
(d) Aedes mosquito
(e) House fly
4. The blood component that has the
highest affinity for oxygen is the (a)
Leucocytes
(b) Erthrocytes
(c) Thrombocytes
(d) Lymphocytes (e) Monocytes

7. Ans: C
Through DNA typing, paternity issues are
very accurately resolved i.e without any
doubts.

8. Ans: D
Bush fallowing refers to the act of leaving a
piece of land for a period (usually years)
without planting on it, and resuming farming
activities at a later time. It increases soil
fertility.

9. Ans: B
The Savannah zones are characterized by the
presence of short scattered trees and sparse
grasses. Also, rainfall is usually low.

10.
11. Ans: C
Let the price of a bottle of malt be ―t‖ and the
price of a bottle of mirinda be ―a‖.
Then t = 3a – (1)
5t + 4a = 380 – (2)
From (1) above:
t = 3a
substitute 3a for t in (2) above 16. Ans: D
5(3a) + 4a + 380
Scoring at least 5 means scoring 5,6,7,8,9 and
15a + 4a = 380
10.
a = 380 19 = 20 Total no of persons = 1+6+1+1=9
substitute 20 for a in (1)
t = 3a
t = 3(20) = 60
therefore, a bottle of malt cost N60 17. Ans: C
Median score = (!!!)th score
!
12. Ans: E n = 15
!".!" !"#$# n + 1 = 15 + 1 = 16
Pr (Bus) = !"!#$!" !" !"!!"#$% (n +1 2) = 16 2 = 8
Pr(Bus) = !"
+ !" = ! The 8th term of 6
!"
13. Ans: C 18. Ans: C
Largest share is the ratio of 8.
Age (x) Freq(f) Sum of ratio = 6+7+8=21
12 5 Amount shared = N10.50
13 12 Largest share = 8 21 x 10.50 = N4.00
14 10
15 3 19. Ans: A
16 3 The power is negative as the value is less than
one.
17 5
18 2 20. Ans: A
!!! th First, determine 40% of 50kg
!"
Median = ( ) term. x 50
!
!""
N =40 = 20kg.
Median = (!"!!)th term Next, find 1 8 of 20kg
!
!" th
= 20.5 term. = 2.5kg (21 kg)
th 2
Looking at the frequency table, the 20.5 This is the mass used for the cake
score is 14. 21. –

14. Ans: B 22. Ans: D


The interval from 1998 to the year 2000 is two This is the main problem. Soaps form an
years. At 2% a year, the total increase will be insoluble scum in hard water
4% 23. Ans: D
New population = 240,000 No of protons corresponds to the position.
!
+ !""" x 240,000
= 249,600 24. Ans: B
Domestic cooking gas is ―LIQUEFIED‖
15. Ans: C PETROLEUM GAS. It is most volatile of all
E the substances listed i.e it most easily changes
into gas.
25. Ans: C
The others burn to release CO2, which is a (c) DNA analysis
major greenhouse gas.

26. Ans: A
Zinc is extensive used in roofing buildings but
zinc is an element, steel, on the other hand, is
an alloy and is extensively used to strength
buildings

27. Ans: C
The earthly impurities are mainly SiO2. The
limestone decomposes as follows:
CaOCO3 → CaO + CO2
CaO then combines with SiO2 to form SLAG:
CaO + SiO2 →CaSiO3
SLAG

28. Ans: A
Sewage refers to excreta produced by man and
is different from solid household waste that is
referred to as refuse. Sewage is biodegradable
in that it can be broken down by
microorganisms.
29. Ans:C
Examples of such elements are: carbon-
diamond and graphite
Phosphorus-white, red and black
Sulphur-rhombic and monoclinic
Tin-white, grey and rhombic

30. Ans: C
Those that absorbs moisture AND
FORMSOLUTION are said to be
DELIQUESECENT
6. The soil type that will be most difficult to plough
in the wet season is the one that is?
(a) Loamy
(b) Clayey
(c) sandy
(d) silty
(e) barkish

7. Paternity disputes can most accurately be resolved


through the use
(a) Finger printing
(b) Blood group typing
(d) Tongue rolling recorded 12 cars and 27 buses. What is the

(e) Physical examination probability that the next vehicle will be a bus?
(a) 4/39 (b) 4/13 (c) 5/13 (d) 6/13 (e) 9/13
8. A farm practice that leads to loss 13. Below is a table showing the ages of
of soil fertility is 40 553 students in Koko

(a) Mixed farming

(b)Bush following
(c)Shifting
cultivation
(d)Continuous
cropping
(e)Desertification

9. In Nigeria, the Guinea savanna belt bordered by the


(a) Mangrove swamps and Sahel savanna
(b) Rain forests and Sudan savanna
(c) beserts and the Sudan
savanna (d)Rain forests and the
deserts
(e) Sahel savanna and guinea savaning
10. Which of the following is used to test for
the presence of lime in a soil sample?
(a) H2S04(aq)
(b)NOOH(aq) (C)HCL(aq)
(d) HNO3(aq)
(e) None of the above

MATHEMATICS
11. A bottle of malt costs three times as much as a
bottle of Mirinda. If 5 bottles of malt and 4 bottles
of Mirinda cost 43 80.00 altogether, how much
does a bottle of malt cost? (a)N90.00
(b) N80.00
(c) N60.00
(d) N40.00
(e) N20.00

12. A petrol attendant recorded the number of vehicles


that bought petrol from him for 3 hours and
Comprehensive 1-ligh School the largest share?
(a) N3.00 (b)N3 .50
Age 12 13 14 16 17 18
Number of 5 12 10 3 3 5
students
Using the above table, calculate the median age of the
students
(a). 13 years 6 months
(b) 14 years 3 months (c) 14 years
(d) 15 years (e) 15 years 3 months
14. If the population of a town was
240,000 in January 1998 and it increased by 2%
each year, what would be the population of the town
in January 2000?
(a) 244,800 (b) 249,600
(c) 249,696 (d)480,000
(e) 480, 600
15. In a class of 80 students, every student had to study
Economics or Geography or both Ecriomics and
Geography. If 65 students studied Economics and 50
studied Geography, how many students studied both
subjects? (a) 15 (b)30 (c)35 (d)45 (e) 50
The table below shows the scores of 15 students in a
Physics test. Use the information to answer
questions 16 and 17

Age 1 2 3 4 5 6 7 8 9 10
Number of 5 12 10 3 3 5
students

16. How many students scored at least 5?


(a) 1
(b) 6
(c) 8
(d)9
(e) 14

17. Find the median score


(a) 1
(b) 5
(c) 6
(d) 7
(e) 8

18. If 3 children share N10.50 among


themselves in a ratio of 6.7:8, how much is
(c) N4.00 (d)N4.50 (e)N-5.00 of crude oil has the highest
19. Express 0.000834 in standard form volatility?
(a) 8.34 x (a) diesel
(b) 8.34 x i0 (b) Domestic cooking gas
(c) 8.34 x (c) Kerosene
(d) 8.34 x
(e) 8.34 x 106

20. A baker used 40% of a 50kg bag of flour. If


1/8 of the amount used was for cake, how many
kilograms of flour was used for cake?
(a) 21/‘? V
(b) 6‖
(c) 15‘ (d) 171‘2
(e) io‖

CHEMISTRY
21. Animal charcoal differs from
wo
od charcoal in that it
(a) Has a dark colour
(b) Has a high carbon content
(c) Is a good absorbent
(d) is crystalline
(e) is rich in phosphate content

22. Detergents are better than soaps for washing in


hard water because?
(a) Hard water contains insoluble calcium salts
(b) They contain bleaching powder (c) They
contain palmitic acid (d) They do not form
insoluble calcium and magnesium salts (e) They
are organic compounds.

23. The position of an element in


the Periodic Table is
determined by
(a) Its atomic radius
(b) Its density
(c) Its relative atomic mass
(d) The number of protons in its atom
(e) The number of neutrons in its atom

24. Which of the following fractions


(d) Lubricating oil without forming a solution is said to be?
(a) Efflorescent
(e) Petrol
(b) eliquescent
25. To reduce global warming by green house‘ gases (c) Hygroscopic
which of the following fuels should countries of the
world should perfect its use as the main source of
energy?
(a) Benzene
(b) Charcoal
(c) Hydrogen
(d) Kerosene
(e) Petrol

26. The alloy used extensively in the


building industry is
(a) Steel
(b) Bronze
Cc) Duralumin
(d) Type metal
(e) Zinc

27. During the extraction of iron , the


limestone fed into the blast furnace produces
CaO which removes
(a) Excess
(b) Un-burnt coke
(c) Earthly impurities
(d) Molten iron
(e) Burnt coke

28. Which of the following pollutants is


biodegradable?
(a) Domestic sewage
(b) Metal scraps
(c) Radioactive waste
(d) Plastic foil
(e) Aluminum foil

29. An element which can exist in two or more


forms in the same physical state exhibits?
(a) Isotropy (b) Structural isomerism (c) Allotropy
(d) Variable valency (e) Metamerism

30. A salt that absorbs moisture from the atmosphere


(d) Insoluble (c) 1963
(e) Effervescent (d) 1961

31. African Union Secretariat is 37. The youngest sovereign state in the World is?
located In………… (a) Eritrea
(a) Abuja (b) Comoros Island
(b) Burkina Faso (c) Cape Verde
(c) Accra (d) Mongolia
(d) Addis Ababo
(e) Cairo

32. A littoral State in Nigeria is


(a) Ekiti
(b) Imo
(c) Anambra
(d) Akwa Ibom
(e) Cross River

33. The State with the highest


number of Local Government
Areas is
(a) Kano
(b) Lagos
(c) Bayelsa
(d) Oyo
(e) Yobe

34. The number of legislators in


Edo State House of Assembly
is?
(a) 18
(b) 2
4
(c)
30
(d)
34
(e)
20

35. GENERAL TEACHING is located


in Local Government Area?
(a) Egor
(b) Ovia South West
(c) Ovia North East
(d) Egor
(e) Ikpoba Okha

36. Nigeria became a Republic in


the year
(a) 1960
(b) 1970
(e) South Sudan. (d) pay for more than the fair price
(e) device a new method of payment
38. The erstwhile governor of Adamawa State is
(a) Tanko El Makura
(b) RamalaYero
(c) Murlala Nyako
(d) Aliyu Babarigida
(e) Sule Lamido

39. Nigeria‘s Honorable Minister of


Education is
(a) Aminu Tambuwal
(b) Aminu Ndalolo
(c) Alhaji Kwankwanso
(d) Alhaji Shekarau
(e) Onyesom Wike

40. A state named after a river in her


domain is?
(a) Kano
(b) Rivers
(c) Baye Isa
(d) Anambra
(e) Ekiti.
Choose from the options lettered A-E one that is the
most appropriate interpretation for each of the following
statements

41. She likes having visitors but she never put herself
out for them. This means she does not
(a) Tolerate them for long
(b) Give up her room for them
(c) Inconvenience herself for their benefits
(d) Stop complaining in their presence
(e) Tell them her secrets

42. The principal‘s warning was like water off a duck‘s


back. This means the warning was
(a) Acceptable
(b) Heeded
(c) Ineffective
(d) Misleading
(e) Reasonable

43. Because of the prevailing fuel scarcity, motorists


have to pay through their nose for one gallon of
petrol. This means that motorists
(a) Have to bribe attendants
(b) Have to queue for long hours
(c) Make advance payments
44. My friend, John is a wet blanket. 49. My uncle has just purchased a Jeep
This means he is a/an person (a) Brand new Japanese light green
(a) Arrogarit
(b) Cool-
headed Cc)
Frustrated
(d) Unenlighteried
(e)Unenthusiastic

45. Awareness has been created on fake drugs and so


it will be gormless for people to still patronize
unauthorized dealers.
(a) angerous
(b) Foolish
(c) Provoking
(d) Unnecessary
(e) Useless

Choose from the options lettered


A-E the one that is nearest in meaning to the underlined
words

46. Human rights activists are mostly inspired by


the desire to protect the people‘ rights
(a) Authorized
(b) Compelled
(c) Constrained
(d) Mandated
(e) Propelled

47. Women enjoy receiving plaudits


from the opposite sex.
(a) Approval
(b) Attention
(c) Giftss
(d) Pummeling
(e) Rebuke

48. The huge debt owed NEPA by customers


has impaired constant electricity supply.
(a) Facilitated
(b) Improved
(c) Prevented
(d) Prohibited
(e) Underlined
Choose from the options lettered A-E
the one that most appropriately
completes each sentence
(b) brand light new green Japanese (b) Could have closed
(c) Japanese brand new light green (c) Must have closed
(d) Japanese light green brand new
(e) light green brand new Japanese

50. Deborah is doing her


(a) Best possible
(b) Possible best
(c) Possible very best
(d) Very best possible
(e) Very possible best

51. They had been earlier informed of


his departure
(a) Hadn‘t they
(b) Hasn‘t they
(c) Haven‘t they
d) Hasn‘t it
(e) Haven‘t it
52. I always buy pepper for my cooking
(a) Grinded
(b) Ground
(c) Grind
(d) Grounded
(e) Grinding
53. John live in a house
(a) Clean new stone-built
(b) Clean stone-built new
(c) New clean stone-built
(d) stone-built clean new
(e) stone-built new clean

54 He gave me a……………
(a) Beautiful small blue metallic jug
(b) Blue metallic small beautiful jug
(c) blue small metallic beautiful jug
d) metallic small beautiful blue jug
(e)small beautiful blue metallic

55. All the offices were locked, so


workers
(a) May have close
(d) Shall have closed (e) None of the above
(e) Would have closed
60. An object placed at the bottom of a well full of clear
water appears closer to the surface due to?
56. The blade of a hoe feels (a) diffraction
colder to touch in the morning (b) Reflection
than the wooden handle (c) Refraction
because the.
(a) The handle contains stored energy in form of heat
(b) The blade is placed at a lower temperature than the handle
(c) Handle is a better
conductor of heat than
the blade
(d) Blade is a better conductor
of heat than the handle
(e) None of the above

57. A drop of petrol makes a finger


colder than a crop of water would
because
(a) A petrol takes its latent heat of
fusion from the finger
(b) Petrol takes its latent heat of
vaporization from the finger
(c) Water has a higher surface tension
than petrol
(d) Water is more viscous than
petrol (e) none of the above

58. Which of the following materials is a


conductor?
(A) plastic (b) Sodium
(c) Wax
(d) Glass
(e) Grease

59. The main purpose of a transformer


in AC radio set is
to…………………
(a) Increase power to the radio
(b) Convert energy from A.C to b.C
(c) Step down the voltage
(d) Step up the voltage
(d)polarization
(e) None of the above 10. -
ANSWERS TO 2014 NURSING PAST – 11. Ans: C
QUESTIONS Let the price of a bottle of malt be ―t‖ and the
price of a bottle of mirinda be ―a‖.
Then t = 3a – (1)
1. Ans. B
5t + 4a = 380 – (2)
Producers are mainly plants. Sometimes, a food
From (1) above:
chain starts with dead fallen leaves; such is referred
t = 3a
to as a ―detritus food chain‖.
substitute 3a for t in (2) above
5(3a) + 4a + 380
2. Ans: D
15a + 4a = 380
Movement ―way‖ from a stimulus I always
described as ―negative‖. Again, taxis is the term we a = 380 19 = 20
use when the WHOLE ORGANISM moves in substitute 20 for a in (1)
response to a stimulus. This, in this case we have t = 3a
NEGATIVE PHOTOTAXISM. t = 3(20) = 60
therefore, a bottle of malt cost N60
3. Ans: D
The aedes mosquito is more implicated in the 12. Ans: E
spread of yellow fever while the anopheles
!".!" !"#$#
mosquito spreads malaria. Pr (Bus) =
!"!#$!" !" !"!!"#$%
!"
4. Ans: D Pr(Bus) = + !" = !
!"!!" !" !"
Anaerobic respiration may be represented as:
C6H12O6 2C02 + 2C2H50H
+ energy ethanol 13. Ans: C
5. Ans: B
Age (x) Freq(f)
Erythrocytes (Red blood cells)contain haemoglobin
that enables them to transport oxygen. Red blood 12 5
cells also transport carbon dioxide. 13 12
14 10
6. Ans: B 15 3
This is due to its high water retention capacity 16 3
which makes it water logged during the wet season. 17 5
18 2

Median = (!!!)th term.


7. Ans: C !
Through DNA typing, paternity issues are N =40
very accurately resolved i.e without any Median = (!"!!)th term
doubts. !"
= 20.5th term.
! th

8. Ans: D Looking at the frequency table, the 20.5


Bush fallowing refers to the act of leaving a score is 14.
piece of land for a period (usually years)
without planting on it, and resuming farming 14. Ans: B
activities at a later time. It increases soil The interval from 1998 to the year 2000 is two
fertility. years. At 2% a year, the total increase will be
4%
9. Ans: B New population = 240,000
!
The Savannah zones are characterized by the + x 240,000
presence of short scattered trees and sparse = 249,600
grasses. Also, rainfall is usually low.
15. Ans: C 24. Ans: B
E Domestic cooking gas is ―LIQUEFIED‖
PETROLEUM GAS. It is most volatile of all the
substances listed i.e it most easily changes into
gas.

25. Ans: C
The others burn to release CO 2, which is a major
greenhouse gas.

26. Ans: A
16. Ans: D Zinc is extensive used in roofing buildings but
Scoring at least 5 means scoring 5,6,7,8,9 and 10. zinc is an element, steel, on the other hand, is an
Total no of persons = 1+6+1+1=9 alloy and is extensively used to strength buildings

17. Ans: C 27. Ans: C


!!!
Median score = ( )th score The earthly impurities are mainly SiO2. The
!
limestone decomposes as follows:
n = 15
n + 1 = 15 + 1 = 16 CaOCO3 → CaO + CO2
CaO then combines with SiO2 to form SLAG:
(n +1 2)th= 16 2 = 8 CaO + SiO2 →CaSiO3
The 8 term of 6 SLAG
18. Ans: C 28. Ans: A
Sewage refers to excreta produced by man and is
Largest share is the ratio of 8. different from solid household waste that is
Sum of ratio = 6+7+8=21 referred to as refuse. Sewage is biodegradable in
Amount shared = N10.50 that it can be broken down by microorganisms.
Largest share = 8 21 x 10.50 = N4.00 29. Ans:C
Examples of such elements are: carbon-diamond
19. Ans: A and graphite
The power is negative as the value is less than one. Phosphorus-white, red and black
Sulphur-rhombic and monoclinic
20. Ans: A Tin-white, grey and rhombic
First, determine 40% of 50kg
!" 30. Ans: C
x 50
!"" Those that absorbs moisture AND
= 20kg.
FORMSOLUTION are said to be
Next, find 1 8 of 20kg DELIQUESECENT
= 2.5kg (21 2kg)
This is the mass used for the cake
21. –

22. Ans: D
This is the main problem. Soaps form an insoluble 31. Ans: D
scum in hard water The AU‘s secretariat is called the AFRICAN
UNION COMMISSION. The AU was
23. Ans: D
No of protons corresponds to the position. established on 26 May 2001 in Addis Ababa,
Ethiopia and launched on July 9, 2002 in
South Africa to replace the organization of
African unity (OAU) which had been
established on 25 May 1983 in Addis Ababa.
The AU has 55 member countries.
32. Ans: D
The word Littoral means ―relating to or Its name is derived from the name of the oma
situated on the shore of the sea or a lake‖. mbala (omambala) river
Littoral states in Nigeria include AKWA-
IBOM, BAYELSA, CROSS-RIVER,
DELTA, EDO, OGUN, and ONDO.

33. Ans: A
Kano state has 44 out of 774 local
government areas in Nigeria. The state with
the fewest local government area is the
FCT(6), viz Abaji, Abuja municipal, Bwari,
Gwa-Gwalada, Kuje, Kwali. Next is Bayelsa
with 9

34. Ans: A
One legislator per local government area.
Edo state has 18 local government areas.

35. Ans: C
The GENERAL TEACHING was established
in the year 1970

36. Ans: D
That was three years after it gained in
dependent (1960)

37. Ans: E
South Sudan became a sovereign state on
July 9, 2011. Magnolia-1992: Cape Verde-
1975, Comoros-1975; Eritrea-2002.

38. Ans: C
He was governor from 29 April 2008 to 15
July 2014.
Sule Lamido-Jigara State
Ramalan Yero-Kaduna State
Tanko El-makura-Nasarawa State
Aliyu Babangida-niger State

39. Ans: D
Mallam Ibrahim Shekarau was minister of
Education between 2014 and 2015 after
Ruqqayat Rufai, who was minister from
April 2010 to September 2013. Shekarau was
succeeded by Mallam Adamu in November
2015 and is the Minister of Education as of
today.

40. Ans: D
maintain
41. Ans: C 52. Ans: B
―To put herself out for somebody‖ means to
make a special effort to do something or
inconvenient herself for their benefits

42. Ans: C
―like‖ water of a duck‘s back‖ means ―the
warning is ineffective‖

43. Ans: D
To pay through one‘s none means ―to
pay more than the fair price‖.

44. Ans: E
A ―wet-blanket‖ means a person who is not
enthusiastic about anything and who stops
other people from enjoying themselves

45. Ans: B
―Gormless‖ means ―stupid or foolish‖
46. Ans: B
“Compelled” which means ―to cause
a particular reaction‖ same as ―inspired‖
47. Ans: A
―praise and approval‖ same as ―plaudits‖
48. Ans: C
―Prevented or damaged or not
functioning normally‖ same as ―impaired‖
Most appropriately completes
49. Ans: B
Because in the order of adjective, determine
(a) come first followed by weight (light)
then age (new), colour (green) followed by
proper adjectives (Japanese) plus the head
noun (jeep)
Note: the options are wrong but option B is
almost correct weight (light) comes before
age (new)
50. Ans: D
―very best possible‖ it is used to indicate
the degree and to emphasize that somebody
is the best, worst of it type

51. Ans: A
―Hadn’t they‖ because in a tag question
a positive statement goes with a negative
tag and the tense of the very must be
―ground‖ because the past and past participle be thorough.
of ―grind‖ is ―ground‖ not ―grinded‖ while 4. Questions booklets are in four options A-D. You are to
―grounded‖ means ―having a sensible and ensure that you shade the option that is on your
realistic attitude to life‖
Most appropriate complete
53. Ans: A
Because in the order of adjectives,
determines (A) comes first followed by
general adjectives (clean) then age (new) and
materials (stone-built) plus the head noun
(house)
54. Ans: A
In the order of adjectives, determiners (A)
comes first followed by general adjectives
(beautiful): then size (small), followed by
colour (blue), then material (metallic) plus
the head noun (jug)
55. Ans: A
―must have closed‖ used to show ―certainty‖
or ―sureness‖ because both ―must‖ and ―have
to‖ are used to say that you are certain about
something (all the offices were locked)
56. Ans: C
This lower the evaporation rate or capacity of
water. Viscosity, on the other hand, describes
more of the ability of the liquid to flow than
to evaporate.
57. Ans: B
Sodium is a metal and metals conduct heat
and electricity.
58. Ans: D
This is a JAMB past question from 2006.
59. Ans: C
This is a JAMB past question from 2008
2013
ENTRANCE EXAMINATION TO SCHOOL
OF NURSING UNIVERSITY OF BENIN
TEACHING HOSPITAL, USELU, BENIN
CITY GENERAL TEACHING -
SATURDAY, 3RD AUGUST, 2013
INSTRUCTION TO CANDIDATES
1. You are to answer the 60 multiple choice type
questions in 80 minutes.
2. A question is represented by a sentence or phrase
followed by stems or suggestions out of which you
are to make choices.
3. The choice made should be indicated by shading the
corresponding box with HR pencit. Any erasure must
question booklet into the appropriate box on the best completes each of the following sentences.
4. His lectures was and so the
OMR form.
students did not understand it
5. SUBMIT BOTH QUESTION
a. indefinite
PAPER AN ORM FORM
UNTAMPEREb WITH ANb
SEPARATELY AT THE ENb OF
THE EXAMINATION
6. MARKS WILL BE tEbUCTE FOR
WRONG ANSWERS
7. THERE SHALL BE NO PENALTY
FOR UNANSWEREb QUESTIONS
8. PHONES, CALCULATORS OR FOUR
FIGURES TABLES ARE NOT
ALLOWED INTO THE HALLS
9. WRITE YOUR NAME BObLY ON THE
QUESTION BOOKLET- SURNAME
FIRST.
10. Each Candidate must sign the space provided
for signature.

In each of the following sentences, there is one


underlined word and one gap. From the
List of words lettered A to E. Choose the one
that is most nearly opposite in meaning to the
underlined word and that will, at the same
time correctly fill the gap in the sentence.
Five years ago the economy of our country was
buoyant but nowadays it has become
a. profitable
b. distressed
c. unbearable
d. low e. comatose
2. Anns efficiency earned her a promotion
while Jones Caused her dismissal
a. lateness
b. capacity
c. incompetence
d. laziness
e. an indictment.
3. In those societies the And
the cultured lived together
a. ignorant
b. rich
c. barbaric
d. arrogant
e. indolent
From the words lettered A to b choose the word that
b. illegible following sentences, choose the word or group that is
c. unintelligible
d. unimpressive
e. exciting

5. Harry cannot do without his Of short stories because


he loves reading
(a)file
(b)library
(c)collection
(d)dictionary
(e) anthology.

6. The river Across the plain


(a) rolled
(b) meandered
(c) trickled
(d) passed
(e) navigated

After each of the following sentences, a list of possible


interpretations is given. Choose the interpretation that
you consider .most appropriate

7. I am given a free hand to run his business. This means


that:
(a) I am allowed to make my own decisions
(b) make my own appointments
(c) assist the workers willingly
(d) alone can choose my executives
(e) I am under no obligation

8. William spent the first three months learning the


ropes. This means that he
(a) pulled all the ropes that should be pulled
(b) learnt what the job demanded
(c) knew what to do
(d) was unwilling to learn
(e) learnt how to tie the rope

9. Our teacher advised us not to go with the crows.


This means that he advised us not to
a. stay too much in the crowd.
b. travel with the crowd
c. have many friend
d. be unduly influenced by others
e. not to sit on the fence
From the word lettered A to E below each of the
nearest in meaning to the underlined word as it is used in 16. A man standing 2 meters from a charcoal fire is
warmed mainly by
the sentence.
a. convection b. radiation
c. refraction
10. John bled profusely as a result of the accident
a. slowly
b. excessively
c. extremely d. abundantly
e. uncontrollably

11. Paakary augmented his income


by selling firewood
a. saved
b. preserved
c. increased
d. economized,
e. elevated
12. In order to keep the peace I had
to retract my statement
a. alter
b. refine
c. withdraw
d. deny
e. rebut
From the words or group of words lettered A to E,
choose the word or group of words that best
completes
each of the following sentences

13. The teacher had hardly turned


his back The class became
noisy;
a. than
b. when
c. then
d. hence
e. that
14. Those boys will have to
blame for not listening to
advice
a. each other
b. ourselves
c. the other
d. themselves
e. oneself
15. You will help me lift this box
a. won‘t you?
b. would you
c. can you
d. could you
e. cant you
d. conduction common generator for their three
e. heat shops and the amount was shared in
17. Which of the following is a secondary
colour
a. Blue
b. Orange
c. Red
d. Green
e. None of the above
18. The counting of currency notes with moist fingers is
based on the principle of
a. Diffusion
b. Cohesion
c. Adhesion
d. Friction
e. Viscosity
19. When cold water is poured on a can containing hot
water, the can collapses because the
a. steam condense and occupies the partial vacuum
in the can
b. external air pressure counter balances the pressure
within the can
c. steam expands to occupy the vacuum remaining in
the can
d. external pressure becomes greater than the
pressure within the can
e. none of the above
20. Convex mirrors are used as driving mirrors because
images formed are
a. erect, virtual and diminished
b. erect, real and diminished
c. erect, virtual and magnified
d. inverted, virtual and diminished
e. none of the above
21. After allowing a discount of 10% on an article, a
seller collected the sum of N2, 700.00. Find the
amount of discount paid.
a. N27.00
b. N270.00 290.00
d. N300.00
e. 2, 400.00

22. If it costs three shopkeepers A, B


and C N918, 000.00 to purchase a
the ratio 1:2:3 respectively. How much would a. 51
b. 34
shopkeeper B pay? c. 17
a. N316, 000
b. N312, 000
c. N309, 000
d. N306, 000
e. N303, 000
23. A body moves at second. Express kilometers
per hour.
a. 25km/h
b. 72km/h
c. 108km/h
d. 215km/h
e. 324km/h

24. Calculate the perimeter of a


square whose area is 100cm2
a. 50cm
b. 40cm
c. 30cm
d. 20cm
e. 10cm

25. Find n if 34n = 100112


a. 5
b. 6
c. 7
d. 8
e. 9

26. What is the remainder when the


median score of the following set of numbers
15, 12, 13, 11, 11, 14, 11 and 13 is divided by
its modal score?
a. 0.1
b. 0.15
c. 1.0
d. 1.5
e. 2.5
27. In a class of 54 students, 37 speak
Yoruba while 20 speak Hausa and 3
speak both languages. How many
students speak neither of the
languages?
d. 3 a. Prof. Osotimehin
e. 0
28. A number is selected at random between 20 and
30 both numbers inclusive. Find the probability
that the number is prime
1
a. /11
5
b. /11
6
c. /11
8
d. /11
9
e. /11

29. Calculate the standard deviation of the following


data; 7, 8, 9, 10, 11, 12, 13.
a. 2
b. 4
c. 10
d. 11
e. 12

30. Simplify 1.25 x 104 2.0 x 10-4


6.25 10
a. 4.0 x i0
b. 5.0 x 102
c. 2.0 x 10_i
d. 5.0 x 10
e. 2.0 x i03
31. The Secretary General of the United Notion is
a. Emeka Anyaoku
b. Banki Moon
c. Boutnus Ghoul
d. Margaret Largades
e. Koffi Anan

32. Economic Community of West African States


has her office in
a. Abidjan
b. Lagos
c. Accra
d. Abuja
e. Cairo
33. Nigeria‘s Minister of Health is
b. br. Muhammed Pate c. Oredo
c. Prof. Onyebuchi d. Ovia South West
d. r. Adenike Grange e. Ovia North East
e. Prof. A.B.C Nwosu

34. A state in Nigeria named after a


river found in her territory is
a. Imo
b. Lagos
c. Ekiti
d. Edo
e. Bayelsa
35. The capital of Lagos State is
a. Lagos
b. Badagry
c. Surulere
d. Victoria Is land
e. Ikeja
36. The state with the fewest
Local Government Councils in
Nigeria is
a. Jigawa
b. Borno
c. Bayelsa
d. Oyo
e. Lagos
37. The number of legislator in the
Upper Chamber of the National
Assemble is
a. 96
b. 109
c. 108
d. 100
e. 360
38. The
number
Councils in
Territory is
a. 6
b. 4
c. 10
d. 12
e. 3
of Local Government the Federal Capital
39. GENERAL TEACHING Teaching Hospital is located in the Local Government Area
a. Egor
b. Ikpoba Okha
40. Nigeria Nation attained independence from a. scaly body
Colonial master in 1960
a. Belgium
b. France
c. United States
d. Germany
e. Britain
41. The vector of the malaria parasite is
a. Male Culex mosquito
b. Female Culex mosquito
c. Female Aedes mosquito
d. Female Anopheles mosquito
e. Male Anopheles mosquito

42. The inheritable characters that are determined by a


gene located only on the X-chromosome is
a. homozygous
b. dominant
c. recessive
d. sex-linked
e. male-linked

43. Lack of space in a population could


lead to an increase in
a. disease rate
b. drought
c. water scarcity
d. birthday rate
e. inheritance rate

44. The part of the mammalian ear


responsible for the maintenance of balance is the
a. perilymph
b. ossicles
c. cochlea
d. pinna
e. membrane

45. The association residing in the ruminant is


a. parasitism
b. predation
c. saprophytism
d. mutualism
e. symbiotism
46. The feature that makes locomotion in
water easy by fish is the
b. slimy body
c. streamlined body Now answer the following questions:
d. lateral line
e. dorsal fin
47. The function of the correcting
lens is to
a. diverge incoming rays
b. converge incoming rays
c. reflect incoming rays
d. screen incoming rays
e. synchronize incoming rays

48. The sheet of muscle that separates the thoracic


and the abdominal cavity is the
a. intercostals muscle
b. pleural membrane
c. pericardium
d. diaphragm
e. lungs

49. At what stage in the life history of a mammal is


the sex of an individual set?
a. At puberty
b. At birth
c. At conception
d. At adolescence
e. At mutation

50. Which of the following organs regulates the levels


of water, salts, hydrogen ions and urea in the
mammalian blood?
a. Bladder
b. Colon
c. Liver
d. Kidney
e. Heart
The next 5 questions below should be answered as
follows:
Shade A if only IV is correct; Shade B if only I and II
are correct; Shade C if only III and IV are correct;
Shade b if only I, II and III are correct; Shade E if I, II,
III and IV are correct
51. Which of the following affect(s) the discharge of
ions during electrolysis?
i. Position of the ion in the
redox series
ii. Concentration of the ions
iii. Nature of electrodes used
iv. Temperature of the solution

52. Potassium and sodium show similar chemical


properties because they
i. belong to the same group in
the periodic table
ii. Have equal number of electrons in their outer-
most shells
iii. Both exits in the +1 oxidation state in their
compounds
iv. Belong to the same period in the periodic table

53. Pitch and asphalt are residual products from the


fractional distillation of petroleum. These
substances are used as
i. protective coatings for road surfaces
ii. Binding agents for roofing materials
iii. Organic solvents
iv. Catalysts for cracking

54. Cause of hardness of water includes the presence of?


i. calcium tetraoxosulphate
ii. Magnesium tetraoxosulphater
iii. Calcium hydrogen trioxocarbonated
iv. Magnesium hydrogen
trioxocarbonate (iv)

55. Which of the following properties is/are


exhibited by catalyst?
a. Altering the rate of a chemical reaction
b. Same mass and composition at the end of the
reaction
c. Causing a change in the equilibrium a. Carbon exhibits allotropy
position of a reversible reaction b. Sulphur exhibits allotropy
d. Starting of a reaction which will normally
nor occur

Each of the next five questions is followed by


five options lettered A to E out of which one is
correct. Give only one answer to each question.
56. An acid present in protein is
a. Lactic acid
b. Amino acid
c. Propanoic acid
d. Palmitic acid
e. Stearic acid

57. Which of the following compound is aromatic?


a. Benezen
b. Cyclobutane
c. Cyclopentane
d. Hexane
e. Ethane

58. A salt which looses mass when exposed to


air is
a. Hygroscopic
b. Deliquescent
c. Efflorescent
d. Fluorescent
e. Effervescent
59. Detergents are better than soap for
laundry because
a. Detergents are synthetic while soaps are not
b. Detergents are more soluble in water
than soap
c. Scum is precipitated when soaps are used
with hard water but not with detergents
d. Soaps from soluble salts with ions
causing hardness of water while
detergents do not
e. Detergents are newer compounds

60. Which of the following statement is


not correct?
c. Chlorine exhibits allotropy 11. Ans: C
―Augmented‖ means to increase: to make larger
d. Hydrogen is a gas
or supplement
e. Argon is a noble gas

ANSWER TO 2013 PAST-QUESTION

1. Ans: D
―Low‖ means in a position comparatively
close t the ground opposite of ―buoyant‖
meaning able to float
2. Ans: C
―Incompetence‖ means inability to perform;
ineptitude opposite of ―efficiency‖ means ―the
extent to which time is well used for intended
task
3. Ans: C
―Barbaric‖ means ―uncultured or uncouth‖
opposite of ―cultured‖ which means civilized,
refined
Best complete
4. Ans: A
―Indefinite‖ means vague or unclear
5. Ans: C
―collection‖ means a set of items of materials
procured or gathered together
6. Ans: D
―passed‖ means an opening, road or track
available for passing
Interpretation
7. Ans: A
―To be given a free hand‖ means I am allowed
to make my own decisions
8. Ans: B
―To learn the rope‖ means ―learnt what the job
demanded‖
9. Ans: D
―Not to go with the crowd‖ means be unduly
influenced by others
Nearest in meaning (Synonyms

10. Ans: B
―Excessively‖ means exceeding the usual
bounds of something same as profusely
extravagantly etc.
first change 100112 to base 10
1x24+ 0x23 + 0x22 +1x21 + 1x20
12. Ans: C
―Retract‖ means to pull back inside same as
withdraw or take back

13. Ans: B
―When‖ because ―had hardly‖ collocates with
―when‖ to denote ―the time‖
14. Ans: D
―Themselves‖ used to refer or reflexive
case of they (those boys)
15. Ans: C
―Would you?‖ a quite polite request.
16. Ans: A
The heat transfer in this case does not
require a medium as in conduction
17. Ans: B
Blue, red and green are the primary colours
18. Ans: D
Moist fingers pose reduced/ less friction
19. Ans: E
20. Ans: C
21. Ans: D
N2,700 = 90% of cost price
∴ N x = 10% (discount paid)
x =!" ! !"## = N 300,000
!"
22.

22 Ans: D
Sum of ratios = 1+2+3 =6
Shopkeepers B pays 2 6 x 918,000

23 Ans: E
To convert from m/s to km/h, e can
simply multiple by 3.6

24 Ans: B
Perimeter = 2 (L x b) = 2(L + L)
But area = L x b = L2 = 100
L = 100 = 10
∴ in km/h, m/s = 90 x 3.6 = 324
Km/h

25 Ans: A
3n + 4 = 16 + 2+
1 !"!! !"
n= = = 5
! !
verification
16+0+0+2+! = 1910 37 Ans: B
The upper chamber (upper house) refers to the
26 Ans: D senate while the lower chamber (lower house)
!"!!"
Median score = = 12.5 refers to the House of Representatives.
!
38 Ans: A
Medal score = 11
39 Ans: E
Difference = 12.5 – 11 = 1.5
40 Ans: E
Nigeria was colonized by the British
27 Ans: E
41 Ans: D The pathogen of the disease is
 54 plasmodine species

42 Ans: D
Such characters include hemophilia and color
37-3+3+20-3+x +54
blindness and are referred to as sex-linked
57_3+x = 54
characters.
54 + x = 54
43 Ans: A Lack of space (overcrowding) leads to
X = 54 + 54 + 0
disease

44 Ans: A
Perilymph is found inside the semicircular canals.
The cochlea is meant for meaning (vibration)
45 Ans: D
46 Ans: C
The reduces the amount of resistances that the fish
28 Ans: A
experiences from the water
Prime numbers between 20 and 30 are 23, and 29
Total numbers between 20 and 30 = 11
! 47 Ans: E
∴ Pr (prime numbers) =
!! This is done either by divergence (for concave
lenses) or by convergence (for convex lenses)
29 Ans: A 48 Ans: D
!!!!!!!"!!!!!"!!"
Mean = = 10 The diaphragm plays a major role in the breathing
!
process.
S.D = = = 4 =2 49 Ans: C
This is the same as saying ―at fertilization‖
30 Ans: A 50 Ans: D
It is named after the Imo River
.. !
35 Ans: E
.! 36 Ans: C
.× ! × .× (!)

!
. × ()
.× !
.×()

= 4 X 10-1- (2)
= 4.0 X 10-1-2 = 4.0 x 10-3
31 Ans: B
32 Ans: D
33 Ans: C
Prof. Onyebuchi Chukwu was health minister as of
the time of this examination
34 Ans: A
51 Ans: D
Temperature of the electrolyte solution does not
affect discharge
52 Ans: D
The main similarity is that found in option B
53 Ans: B
Surfacing of roads and mending of roofs are two
important applications of asphalt and pitch
54 Ans: E
It must be mentioned; however, that magnesium
hydrogen trioxocarbonate (iv) does not readily
exist in solution/ cause hardness. Meanwhile,the
sulphates mentioned cause permanent hardness

55 None of the option is correct. Only statement I


and IV are correct catalysts do not affect
equilibrium
position. Catalysts may change physically but not
chemically

56 Ans: B
Amino acids are the building blocks of protein.
Lactic acid is found in milk
Palmitic and stearic acids are fatty acids and are
thus found in fats

57 Ans: A
Cyclobutane and cyclopentane are cyclic aliphatic
hydrocarbons
Hexane and ethane are acyclic aliphatic
hydrocarbons
Benzens is an aromatic hydrocarbon

58 Ans: C
Efflorescent substance such as Na 2CO3. 10H2O
lose all or part of their water of crystallization
when exposed to air or when heated. They usually
change in physical appearance too.

59 Ans: C
Scum is the name given to any of the misoluole
salts precipitated when the calcium and/or
magnesium ions is hard water come in contact
with soap.

60 Ans: C
Chlorine exhibits isotopy
!" !"
( ) but not allotropy
!" !"
Allotropes of:
- Carbon include diamond and graphrite
- Sulphur include rhombic and manoclinci
sulphur
ENTRANCE EXAMINATION TO SCHOOL the true length of the room?
OF NURSING UNIVERSITY OF a. 3.78meters
BENIN TEACHING HOSPITAL, b. 3. 80 metres
USELU, BENIN CITY,
GENERAL TEACHING PAST QUESTIONS,
INSTRUCTION TO CANDIDATES

A sales girl gave a change N1.15k to a


customer instead of N1.25k. Calculate
her percentage error
a 10%
b 8.7%
c. 8.0%
d. 2.4%
e. 0.01/0

2. If three children share N10.50k among themselves in


the ratio of G:7:8, how much is the largest what?
a. N3.00
b. N3.50
c. N4.0O
d. N4.50%
e. N5.00%
3. By selling some crate of soft drinks for N600,00 a
dealer makes a profit of 50%. How much did the
dealer pay for the drinks?
a. N1200.00
b. N900.00
c. N450.00
d. N400.00
e. N200.00
4. A trader made a loss of 15% when
selling an article. Find the ratio,
selling price; cost price
a. 3:20
b. 3:17
c. 17:20
d. 20:23
e. N15:100
5. A man bought a television set on hire purchase for
N25,000, out of which he paid N10,000. If he is
allowed to pay the balance in 8 equal installments
find the value of each installment
a. N1250
b. N1578
c. N1875
d. N2250
A student measured the length of a room obtained the
measurement of 3.99metres. If the percentage error
of his measurement was 5% and his own
measurement was smaller than the length, what is
c. 4.18 metres b. On
d. 4.20 metres e. 4.788metres c. In

7. Amina has mangoes she ate 3 and shared the


remainder equally with her brother Uche. Each
had at least 10. Which of the following
inequalities represent the statement above?
a. M-3
2 <10
b. M-3
2 >10
c. M-3
2 >10
d. M-3
2 <10
e. M-3
2 = 10

8. A car moves at an average speed of


30km/h, how long does it take to cover 200meter
a. 2.4 seconds
b. 24 second
c. 14 seconds
d. 240 seconds
e. 1 second

9. What is the smaller value of x


for which X2 - 3X + 2 0?
a. 1
b. 2
c. 3
d. 4
e. 5

10. Which of the following is necessarily sufficient


for construction of a triangle?
a. Three sides
b. Two sides and an included angle
c. Two sides and a right angle
d. Two sides and an angle
e. Two angles and a side

Complete each of the following sentences by choosing


the options that
most suitably fill the spares
11. Amos must keep his speed if he hopes to succeed
a. At
d. Up 17. Mr. Ama had an….......Training yesterday as he was
e. On with preparing for the following match

12. My mother the shell of the hard boiled egg much


quicker
than I thought
a. Bust
b. Snapped
c. Shattered
d. Cracked
e. Fractured

13. Our bus traveled at a high that day


a. Speed
b. Rapidity
c. Crate
d. Quickness
e. Acceleration

14. Johns new house is very for the office as he can set
there in six minutes
a. ConvenIent
b. Available
c. Suitable
d. Pleasant
e. Comfortable

15. Most people can no longer accommodate his long


Story
a. Irritable
b. Boring
c. Weary
d. Tiding
e. Annoying

16. The highlife music aroused an


feeling of homesickness in Jonathan
a. Intending
b. Intensive
C. Intense
d. Intended
e. Intentional
a. Intensive
b. Intense 22. The auditor‘s account of the entitles made in the
c. Intended ledger caused the latter no embarrassment. From this
d. Intentional we that the ledger clerk was:
e. Intending a. Not embarrassed at all

18. We visited the school for in India


a. beafs
b. The deaf
c. The deafs
d. beafness
e. The deafness

19. The panel sat all through the night to resolve the
issue but the peace meeting In confusion
a. Broke through
b. Broke out
c. Broke down
d. Broke up
e. Broke about

20. The principal made it clear that of us must


endeavour to pass the examination
a. All and everyone
b. Each someone
c. Each and everyone
d. Each everyone
e. Everyone
After each of these sentences, a list of suggested
interpretations of all or part of the sentence is given.
Choose the interpretation you consider most
appropriate for each sentence and record your answer
accordingly

21. The politician stood before the crowed and said


in a forceful voice
―Am I the sort of man to deceive
you in a matter of such
importance?
a. Seeking the answer to this question
b. Apologizing for having deceived the crowd
c. Entreating the crowd
not to let themselves be
deceived
d. Really saying that he was not such a man at all
b. Rather pleased
c. Very happy
d. Greatly embarrassed
e. Fairly embarrassed

23. The news that Vice-Chancellor was


sacked sounded too fantastic to
believed. This means that the news
a. Was actually true
b. Was so obviously untrue
c. Was very strange and difficult After each of the following sentences, a list of possible
to believed interpretations of all part of the sentence is given.
d. Was only partially true Choose which interpretation you consider appropriate
for each sentence from the list of multiple choices
e. Could be believed
attached.

26. Because of the present inflation in Nigeria the


Nigerians workers find it difficult to
a. Keep two jobs at a time
b. Live within their means
c.
27. The policeman ran the murderer to earth after two
months of work. This statement means that the
police:
24. Rather than be humiliated in public Mr. Akaraka a. Struck the murderer
decided to fight. From this statement we can infer b. is covered the murderer by
that: searching
a. He was reluctant to fight c. an up to the murderer after long chase
b. He was not going to fight, whatever happened d. Found the murder where he
c. He was challenged but refused to fight public was hiding under-ground
d. He was determined to fight, whatever happened e. Shot the murderer dead while
e. He fought but would have pref erred not do so. trying to get away.

25. The students should never have followed Mr. Ak‘s 28. What you have said is quite beside the
advice if they had not known his reputation. This This means that what you said point.
means that the students a. Very important
a. Knew what people said about b. Another good point
Mr. Aka, and so refused to obey him c. An obvious
b. Trusted Mr. Aka very much but not need any d. Very near the point
e. Completely irrelevant
advice
c. Had heard a great deal about Mr. Aka, but were 29. John‘s action amounted to sitting on the fence from
disappointed when they heard him speak. this we know that John was certainly
d. Trusted Mr. Aka and took his advice a. disloyal to this friend
b. Cowardly
e. id not know Mr. Aka reputation
c. Undecided on which side to join
d. Treacherous
e. Prevented from taking a life decision
30. Reasonable people usually steer a middle course in 35. A painter charges N12 an hour while his son charges
all their undertakings. In this statement, the N6 an hour. If father and son work the same amount
Italicized words means of time together on a job, how many hours does each
a. work very hard of them work if their combined charge for labour is
b. Save much time N108?
c. Do not waste money a. 6
d. Act with moderation b. 8
e. Travel quite slowly c. 9
d. 12
31. Chris finishes the first half of an exam in two-third e. 18
the time it takes him to finish the second half. If they
exam takes him an hour, how many minutes does he 36. If a man earns N200 for his first 40 hours of work in
spend on the first half of the exam? a week and is then paid one-and half times his
regular hourly rate for any additional hours, how
32. If the ratio of boys to girls in a class is 5 to 3 which many hours must he work to make N230 in a week?
of the following could not be the number of students a. 4
class? b. 5
a. 32 c. 6
b. 36 d. 44
c. 40 e. 45
d. 48
e. 56

33. There are enough peanuts in a bag to give 12 peanuts


to each of 20 children, with no peanuts left over
children do not want any peanuts, how many peanuts
can be given to each of the others?
a. 20 37. At a certain school, if the ratio of teachers to students
is 1 to 10, which of the following could be the total
b. 15 number of teachers and students?
c. 15 a. 100
d. 40 b. 121
c. 144
d. 222
e. 1,011

38. After 21 kids were added to a class,


there were 4 times as many students
as before. How many kids were in the
class before the addition?
a. 3
34. A footballer practices one hour a day from Monday b. 7
through Friday. How many hours must he practice c. -
Saturday in order to have an average (arithmetic d. 14
mean) of two hours a day for the six days period? e. 17
a. 5
39. A survey finds that 80 percent of the apartments in
b. 7 city G have smoke alarms installed. Of these 20
c. 6 percent have smoke alarms that are not working.
d. 8 What percent of the apartment in City G were found
e. 12 to have working smoke alarm?
a. 60%
b. 64%
c. 662/3 40 to what number?
d. 70% a. 3
e. 72%

40. A candidate average 72 on-5 tests. If the lowest score


is dropped, the average rises to 84. What is the
lowest score?
a. 18
b. 24
c. 32
d. 43
e. 48

41. A supply of sugar lasts for 90 days. If its use is


increased by 50 percent, how many days would the
same amount of sugar last?
a. 15
b. 20
c. 25
d. 30
e. 45

42. The product of three positive


integers is 300. If one of them is 5,
what is the least possible value of the
sum of the other two?
a. 16
b. 17
c. 19
d. 23
e. 32

43. The population of a country doubled


every 10 years from 1960 to1990.
What was the percent increase in
population during this time?
a. 200%
b. 300%
c. 700%
d. 800%
e. 100%

44. The percentage decrease from 12 to 9


is equal to the percent decrease from
b. 10 e. 100
c. 25
d. 30 50. The erstwhile secretary to government of Nigeria is
e. 36 a. Yayale Ahmed
b. Ahmed Joda

45. The highest number of medals in the last


Olympics held in Beijing China was won by
a. China
b. USA
c. Russia
d. Spain
e. France

46. The African country with the


highest medal haul in the last
Olympic is
a. Ethiopia
b. Nigeria
c. Kenya
d. South Africa
e. Ghana

47. Earthquake occurring beneath the sea is called


a. Tormado
b. Cyclone
c. Tsunami
d. Landslide
e. Sand dunes

48. The capital of Jigawa state is


a. Jalingo
b. Maiduguri
c. Dutse
d. Gombe
e. Kano

49. The number of senatorial seats


in Nigeria is
a. 109
b. 108
c.
36
d.
774
c. Ufot Ekaette b. Spain
d. Babagana Kingible c. France
e. Shehu Musa
51. The current Chief of defence staff in
Nigeria is
a. Paul tike
b. Ogomudia S.
c. Azazi
d. Agwai
e. L. Yusuf

52. Along the West Africa Coast, the


Greewich Merdian passes through
a. Nigeria
b. Togo
c. Gambia
d. Ghana
e. Senegal

53. The secretarial of the Economic Community of West


Africa States (ECO WAS) is located in
a. Cote de voir
b. Ghana
c. Nigeria
d. Mail
e. MauritanIa

54. One of the following Africa countries


is under a military rule
a. Rwanda
b. Liberia
c. Sudan
d. Mauritania
e. The Gambia

55. The capital of Rwanda is


a. Nouakchott
b. Kigali
c. Kampala
d. Ouagadougou
e. ………………

56. The current European football


champion is
a. Italy
d. Britain Intense means acute, strong, or vehement, as
senstJos1 feelings or
e. Belgium
emotion, or extreme degree

57. The next Olympic is stated for


a. Scot land
b. Britain
c. France
d. Japan
e. South Korea

58. The Littoral states in Nigeria include


a. Ekiti State
b. Edo State
c. Akwa-Ibom State
d. Kaduna State
e. Benue State.

59. The following except one had


served as capital of Nigeria
a. Calabar
b. Lagos
c. Zungeru
d. Lokoja
e. Kaduna
ANSWER TO 2012 VITH NURSING PAST
QUESTION
11. Ans: D
―Keep up‘ means to make something stay at a
high level
12. Ans: D
―Cracked‖ means ―to break open or into
place j1to?iece
13, Ans: A
speedu ieans the rate at which somebody or thing
moves or travels
14 Ans: C
―SuitabJe‘ means ―in a way that is right or
appropriate for a particular purpose

15. Ans: B
―Boring‖ means ―uninteresting‖ and tiring
16. Ans: C
30. Ans: b
17. Ans: D
―Intentional‘ means done with, intention or on purpose;
intended
18 Ans: B
―The deafu means those classified as partially or
wholly lacking or deprived of the sense f hearing,
unable to hear

19, Ans: C
―Broke down‖ means to stop working or function
properly or stop suddenly

20. Ans C
―Each and everyone‖ means each without
exception; used for emphasis

21, Ans:
―Really saying that he was not such
a man at all‖
22. Ans: D
―No little embarrassment‖ means ―greatly
embarrassed‖
23. Ans: C
―Sounded too fantastic to be believed‖ means was
very strange and difficult to believed
24. Ans: E
―He fought but would have preferred not do so‖
25. Ans: E
―If they had not known, his reputation‖ means they
did not know Mr. Aka reputation
26. Not complete
27. Ans: B
―Can the murderer to earth means discovered the
murderer by searching‖
28. Ans: E.
―Quite beside the point‖ means ―completely
irrelevant‖
29. Ans: C
―To seat on the fence‖ means ―undecided which side
to join‖
―To steer a middle course in all their A. Speed
B. bensity
undertakings‖ means ―to act with moderation‖

2011
GENERAL NURSING PAST QUESTIONS
AND ANSWERS
- SATURDAY 16TH JULY 2011
INSTRUCTIONS TO
CANDIDATES

1. You are to answer the 60 multiple choice type


question in 60 minutes.
2. A question is represented by a sentence or phrase
followed by stems or suggestions out of which
You are to make choices
3. The choice made should be indicated by shading the
corresponding box with HB pencil. Any erasure
must be thorough.
4. Question booklets are in four options A— b. You
are to ensure that your shade the option that is on
your question booklet into the appropriate box on
the OMR form
5. Your are WRITE and SHAbE in your NAMES,
NUMBER and PAPER OPTION (ITEM 5 ON
THE OMR FORM) in spaces provided. In filling
your name, please a space be allowed between
SURNAME, MIbbLE and FIRST names.
6. Your examination number should be filled as
follows: S0N000078 for a candidate whose
number is 784 on the list and SON 0000028 for a
candidate whose number is 28 on the list.
Please ignore items 6,7,8 on the OMR forms
7. SUBMIT BOTH QUESTION PAPER And OMR
FORM Unhampered WITH And SEPARATELY
AT THE END OF THE
EXAMINATION
8. MARKS WILL BE deducted FOR
WRONG ANSWERS
9. THERE SHALL BE NO PENALTY
FOR UNAN5WERE QUESTIONS
10. PHONES, CALCULATIONS OR
FOUR FIGURES TABLE ARE NOT
ALLOWED INTO THE HALLS
11. WRITE YOUR NAME BOLDLY ON THE
QUESTION BOOKLET-SURNAE FIRST
OPTION C

1. Which of the following is a fundamental quantity?


C. Length E. None of the above
b. Impulse
E. Energy 7. The reaction of vegetable oil with a
solution of wood ash is
2. A body moves with a constant speed but has an A. Saponification
acceleration this is possible if it B. Neutralization
A. Moves in a straight line C. Hydrogenation
B. Move in circle b. Eterification
C. Is oscillating E. None of the above
b. Is in equilibrium
E. Has a varying acceleration. 8. When excess ethane is shaken with acidified KMnO4
Solution, the product obtained is
3. Power is defined as the A. Ethane
A. Capacity to exert a force B. Ethanal
B. Product of force and time C. Ethane-12-diol
C. Product of force and distance b. Ethanoic acid
b. Ability to do work E. None of the above
E. Energy expended per unit time

4. Which of the following is not a mechanical wave?


A. Wave propagated in stretched string
B. Wave in closed pipe
C. Radio waves
b. Water waves
E. Sound waves. 9. When a sample of water was boiled, it lathered more
readily with soap it can be concluded that the sample
most
likely contained
A.Magnesium tetraoxosulphate
(VI)
B. Suspended solids
C. Organic impurities
D. Calcium
5. What type of motion does the skin of a talking drum hydrogentrioxocarbonate (VI)
perform when it is being struck with the drum stick? E. None of the above
A. Random
B. Rotational 10. What are the two gases with the
C. Vibratory formation of acid rain?
b. Translational A. CO2 arid HCI
E. Circular B. CO2 and N2
C. 502 and NO2 HCI and 502
6. A reducing agent is expected to ……… E. None of the above
A. Decolourize acidified KMnO4
solution C. Liberate b. Liberate CO2 from NaHCO3
B. becolourize acidified Fe504 Cl2 from a
chloride
!
11. Express as a percentage,
!"
correct to identify
A. 27%
place.
B. 37%
C. 27.1%
D. 36.8% B. 5/16
E. None of the above C. 8/13

12. A car moves at an average speed of


30km1 how long does it take to cover 200 meters?
A. 2.4sec
B. 24sec
D. l44sec
C. 24Osec
E. None of the above

13. A man bought a television set on hire purchase for


Nb, 000, out of which he paid N 10,000. If he is
allowed to pay the balance in eight equal
installments, find the value of each installment.
A N.1250
B. N1578
C. N 1875
E. N 3125

14. In a case of 80 students, every student studies


Economics, Geography or both. If 65 students stud
Economics, and 50 study Geography, how many
study both subjects?
A. 15
B. 30
C. 17 ½
D. 45.
E. None of the above.

15. A man is four times as old as his son.


The difference between their age is
36s. Find the sum of their ages
A. 45 years
B. 48 years
C. 60 years
d. 74 years
E. None of the above.

16. Simplify (3/4- 1/3) x 41/3 ÷ 31/4


A. 5/9
D. 16/5 cameral Parliamentary
E. None of the above 22. The newest sovereign nation in the
world is
A. Egypt
B. Northern Sudan
17. Three men, Bedu, Bakre arid Kofi shared N 500 in
the ration 3:2:x respectively. If Bedu‘s share is
N150, find the value of x
A. 1
B. 4
C. 5
D. 6
None of the above

18. A school girl spends 1/4 of her pocket money on


books and 1/3 on dress. What fraction remains?
A.
5
/6
B.
7
/12
c.
5
/12
D.
1
/6
E. None of the above

19. There are m boys and 12 girls in a class. What is


the probability of selecting at random a girl from
the class?
A. M/12
B. 12/M
C. 12/
MD
MM
E. None of the above.

20. Simplify 71/2 - (21/2 + 3) + 161/2 and correct


your answer to the nearest whole number
A. 33
B. 8
C. 7
E. None of the above

21. Nigeria‘s national legislature is in structure


A. Unicameral
B. Bicameral
C. Executive Tri-
C. Comoros Island Southern Sudan E. Sokoto
E. Eritrea
29. The only State of the Nation
23. The speaker of the House of contro lied by Congress forrn
lepresentative in Nigeria is Progressive Charge (CPC) is
A. bimeji Lawal A. Kastina
B. Mr. U. Igbe B. Benue
C. Mrs. Felicia Etteh C. Zamfara
b. Alhaji Tambuwal D. Nasarawa
E. Bellow Masar. E. Niger,

24. State named after rivers in their 30. Edo State is currently represented at
domain include the Federal Cabinet by
A. Ekiti A. Prof. thonvbere
B. Lagos B. Chief Mike Ogiadomhe
C. Bayesa C. Mike Onoleimemhen Engr. Chris
b. Imo Ogiemwonyi
Kano E. Senator Uzamere.

25. The headquarter of Egho Local


Government Area is
A. Uselu
B. Idogbo
C. Ugbekuu Aduwawa
E. GENERAL City

26. The 2011 FIFA Organized Female


Football World Cup competition is held in
which of the following countries? 31. Oxygenated blood is pumped to the
A. France entire body from the part
B. Ghana A. Left ventricle
C. Nigeria Germany B. Right ventricle
E. None of the above C. Right atrium
b. Left atrium
E. None of the above
27. Nigeria become a republic in the year
A. 1960 32. Which of the following has the most
B. 1959 primitive respiration system?
C. 1963 1970 A. Snail
E. 1964 B. Fish
C. Insect Mouse
28. The current Vice President, Alhaji C. Adamawa Kaduna
Namadi Sambo was previously the
government of ... State
A. Kano
B. Jigawa
E. None of the above

33. The cell of an onion bulb can be


differentiated from a check cell
by the presence of
a. Plasma lemma
b. Chloroplast c. cell wall
d. Nucleus e. none of the above
e. none of the above

34. A plant parenchyma cell also acts as a


supporting tissue when it
a. becomes flaccid
b. contains crystal
c. becomes turgid
d. is pigmented
e. none of the above
40. Which of the following describes the
35. The major consequence of bush burning in an sequence of blood flow from the heart to
ecosystem is a tissue?
a. the loss of water absorbing ability of the soil a. Heart—artery—arteriole—tissue
b. the loss of biological diversity b. Heart —vein venule— —‗tissue
c. a decrease in animal population c. Heart—k venule —‗ vein—‘
d. an increase in soil fertility —tissue
e. none of the above d. Heart—arteriole —of artery— tissue
36. A characteristic exhibited by all living e. none of the above
organisms is
a. sexual reproduction c. Yeast
b. aerobic respiration c. the ability to move from d. Spirogyra
one place to another
d. the ability to remove unwanted substances.
e. none of the above

37. In a cell, the genes are carried by


a. nuclear membranes
b. chromatic threads
c. lysosomes
d. mitochondria
e. none of the above

38. The first terrestrial vertebrates


evolved from
a. Pisces
b. reptihon
c. amphibian
d. mammalian
e. none of the above

39. Which of the following is an example


of carnivorous plant?
a. Hydra
b. Bladderwort
41. The enzymes of the glycolytic pathway
are located in the
a. mitochondria
b. gastric juice
c. plasma
d. cytoplasm
e. none of the above

42. A marine protozoan is likely to have no


contractile vacuole mainly because the
cytoplasm is
a. isotonic to sea water b. hypotonic to
sea
water
c. hypertqnic to sea water
d. impervious to sea water
e. none of the above

43. The major, cause of global


warming is the
a. burning of fossil fuel
b. construction of dams
c. use of electricity
d. exploration of space
e. none of the above
e. convinced

44. A woman with the ability to roll her tongue


(Tt) marries a man who cannot roll his tongue (tt). What
is the probability of each of their children being a
tongue roller?
a. 100%
b. 75%
c. 50%
d. 25%
e. none of the above

45. The part of a domestic fowl responsible


for preventing heat loss is the
a. filoplume
b. contour feather c. down feather
d. quill
e. none of the above
From the words lettered A to below each of the
following sentences, choose the word that is nearest in
meaning to the underlined word as it is used in the
sentence.

46. The Journalist refused to divulge the


source of his information
a. confirm
b. inform
c. disclose
d. extract
e. affirm

47. As soon as Shehu realized that he has


made a mistake, he did everything to rectify it.
a. destroy
b. erase c. eradicate
d. relieve e. correct

48. Ada was skeptical about the story


ifeoma told her.
a. ignorant
b. doubtful
c. confused
d. happy
49. Flight WT509 from Lagos terminates in Maiduguri.
55. Science and Technology
a. closes
many African countries have not made
b. concludes
c. lands
d. ends
e. ceases

50. The prison officials usually torture the inmates


a. lynch b. murder c. torment d. destroy
e. humiliate
From the words or group of words lettered A
to D, choose the word or group of words that
best Completes each, of the following
sentences.

51. To be alive after such a


serious accident a
celebration
a. is called for
b. calls for
c. has been called for
d. call for
e. none of the above

52. She does not remember ever a tiger


a. to see
b. to have seen
C. seeing
d. having to see
e. none of the above

53. The Commissioner for Education with


his secretary expected yesterday
a. are
b. was
c. were
d. is
e. none of the above

54. But for the timely intervention of


the police, the kidnapper by the
angry mob
a. will be lynched
b. would have been lynched
c. would be lynched
d. will have been lynched
e. none of the above
significant progress since their (c) Shore
independence. (d) Bed
a. with regards to (c) None of the above
b. in respect to
c. as far as 60.
d. with regard to (a) Usable
(b) Saleable
In the following passages, the numbered (c) commercial
gaps indicate missing words. Against each (d) practical
number in (e) None of the above
The list below each passage, five options,
are offered in columns lettered A to 6.
Choose the
Word that is the most suitable to fill the
numbered gap in the passages.
Oil is perhaps the most sought after of all
_11_ however a major difficulty in its ANSWERS TO 2011 NURSING
_12_ is that oil _13_ are very often below QUESTIONS
he sea _14_ As a result, the search for oil
1. Ans: C
is very expensive, and unless oil is found
Other fundamental quantities are time
there in _15_ quantities the companies or
and mass
government involved are liable to lose
heavily. 2. Ans: B
56. 3. Ans: E
(a) Products Mathematically; Powers time
(b) Commodities 4. Ans: C
(c) Liquids Radio waves are electromagnetic waves.
(d) Natural resources 6. Ans: C
(e) None of the above Vocational motion is also called

57 (a) Surface
(a) Manufacture (b) Shelf
(b) Exploration
(c) Research
(d) Investigation
(e) None of the above

58.
a) Fields
(b) Plants
(c) Mines
(d) Holes
(e) None of the above

59.
: B
O
S The
CI process gives
L soap as
L
A product. Wood ash is a source of potassium
T 9. Ans: C
O Alkaline
R kmn°4
Y would give
M an alkanol
O
TI 10. Ans: D
O Hardne
N ss removed by
7. A
boiling is
n
temporary
s and
must
: have been due to a hydrogentrioxocarbonate
11. Ans: C
A
Ac
8
id rain is
O
due
2 to the
oxides of
i
nitrogen,
s
sulphur
a
and
n
carbon.
e a percentage, 7/
As 19
x
becomes
a
m
p
l
e

i
n

t
h
i
s

r
e
g
a
r
d
8. A
n
s
7
/19 x 100 = 36.8% Fr a c t i o n sp e n t = 1 / 4 (o r 3 / 1 2 )+ 1 / 3 (0 r
! !
!" !"
12. Speed = distance/time
F ra c t i o n l e f t
Time= distance/speed
Time = 200/30 = 6.67 sec.
19. An s : c
To t a l n o . o f s t u d e n t s = m + 1 2
12
Pr ( s e l e c t i n g a g i r l )
13. As he already paid HI O, 000, he is to balance H m+12
15,000 over eight (8) installments
Each installment N1875
14. Ans: E 20. A ns : E
Us i n g B ODM AS
=18.5
=19(nearest whole no 33/2

21. An s : B
It c o n s i s t s o f a n u p p e r h o u s e (o f
65-x+x+50-x+ 80 s en at e) an d a l o w er h o u s e ( o f
115-x=80 re p re s e n t a t i v e )
X=115-80=35 22. An s : D
23. An s : D
24. An s : D
15. Ans: C
Let the man's age be x and his son's age y. Then x= 4y
Also x-y=36 It i s n a m e d a f t e r t h e Im o R i v e r. L a g o s
is n a m e d a f te r its m a n y w a te r s .
This means that 4y-y=36 3y=36 Y=Y= 12yrs
25. An s : A
Man's age = x:4y=4xl2=48yrs
26. An s : D
Sum of their ages= x +y = 48 + 12+
27. An s : c
60yrs

Ho we v e r , s h e b e c a m e i n d e p e n d e n t i n 1 9 6 0
16. Ans: A
28. An s : D
Using BODMAS
29.---------
(3/4 – 1/3) x 41/3 ÷ 3 ¼ Becomes
(9 124 - 14 12) x 13 3 ÷ 13 4
! !" !
= !"
!" ! !" !
! ! x 30. An s : c
= !" !
!" !
31. An s : A
Fr o m th e le f t v e n tr ic le , b lo o d is
pum pe d out of t he he a r t t hr o u g h t he
la r g e s t a r te r y in th e b o d y c a lle d th e
AOR T A
32. An s : A
Th e y a r e t h e m o s t p r i m i t i v e o r g a n i s m
17. Ans:c on t he l i s t
See Q. 11 of ye a r 2009
18. An s : c 33. An s : A
On i o n b u l b c e l l a n d c h e c k c e l l a r e
pl a nt a nd a ni m a l c e l l s r e s pe c t i ve l y.
34. An s : C
Tu r g id m e a n s f lu id - fi l l e d
35. An s : B , C
Ma n y s o i l o r g a n i s m s d i e s o t h a t t h e
an i m al p o p u l at i o n d ecr eas es an d
bi ol ogi c a l di ve r s i t y i s e nda nge r e d
36. An s : E

37. An s : B
Th e s e c h r o m a t i n t h r e a d s r e p r e s e n t t h e
ch r o m o s o m es
38. An s : c
39. An s : B
Ot h e r c a r n i v o r o u s ( o r i n s e c t i v o r o u s )
pl a nt s in c lu d e b u tte r w o r t, V e n u s
fl y t r a p , p i t c h e r p l a n t a n d s u n d e w
(d ro s e ra ). T h e y p ro d u c e t h e i r o w n f o o d
but r e qui r e i ns e c t s f or t he i r ni t r oge n
su p p l y
40. An s : A
41. An s : 0
Th o s e o f k r e b ' s c y c l e a r e l o c a t e d i n t h e
mi t o c h o n d r i o n
42. Ans: A "Is" because in an accompaintment concord, the
Contractive vacuoles are usually for commissioner for education is the focus of the topic
osmoregulation (i.e. salt and water balance). and not the secretary who accompanied him,
therefore, the verb must be singular
44. Ans: C

54. Ans: B
"Would have been lynched" meaning the police
came on time, therefore the kidnapper was not
lynched
55. Ans: D
"With regard to" because regards collocates with
Non- Tongue rollers Pr (tongue rollers) = 2/4 = ½ "AS" while regard collocates with "with"
50% 56. Ans: D."Natural resources" means
Ans: A 57. Ans: B
Fossil fuels release oxides of carbon (C0 2 in "Explorations" means "the act of travelling through
particular), which is a a place in order to find out about it or look for
45. Ans: C 53. Ans: D
46. Ans: C
"Disclose" same as "divulge" which means to
give somebody information that is supposed to be
secret
47. Ans: E
"Correct" same as "rectify" which means to put
right something that is wrong
48. Ans: B
"Doubtful" same as "skeptical" which means
"having doubt that a claim or statement is true or
that something will happen
49. Ans: D
"Ends" same as "terminates" which means to end;
to make something end
50. Ans: C
"Torment" same as "torture" means the act of
causing somebody severe pain in order to punish
them or make them say or do something
BEST COMPLETE
51. Ans: B "Calls for" means "the need for
something"
52. Ans: C
"Seeing" because the "ing" form of the verb
(gerund) attracts the following words: remember,
imagine, regret, busy, fear, mind, resent, miss,
involve, detest etc.
something
58. Ans: A
"Field" means a large area of land covered with the
thing mentioned
59. Ans: A "Surface" means the outside or
top layer of something
60. Ans: C "Commercial" means connecting with
buying and selling of goods and services or making or
2010 b. Butterflies
ENTRANCE EXAMINATION TO SCHOOL c. Cockroaches
OF NURSING UNIVERSITY d. Termites
OF BENIN TEACHING e. Mosquitoes
HOSPITAL, USELU, BENIN
CITY
GENERAL NURSING PAST QUESTIONS AND
ANSWERS
1. Which of the following elements is required in
large amount by plants
a. Molybdenum
b. Boron
c. Copper,
d. Phosphorus
e. Calcium

2. The products of respiration are


a. Nitrogen and water
b. Nitrogen and carbon dioxide
c. Water and oxygen
d. Carbon dioxide and water
e. Carbon monoxide and water

3. Which of the following features enables the


tapeworm to live
successfully in the small intestine of
human
a. Hooks and suckers for strong attachment
b. A long neck for forming young proglottis
c. The absence of segmentation appendages
d. Possession of flame cells for excretion
e. Proboscis and suckers

4. Plants found in the aquatic habitat are generally


referred to as
a. Epiphytes
b. Halophytes
c. Xerophytes
d. Hydrophytes
e. Saprophytes

5. Which of the following organisms


exhibit division of labour ?
a. Houseflies
6. Correct 0.04945 to two 10. Find, correct to two decimal places,
significant figures. the mean of 9, 13, 16, 17, 19, 23, 24.
a. 0.040 a. 23.00
b. 0.049 b. 17.29
c. 0.050 c. 16.50
d. 0.49 d. 16.33
e. 0.89 e. 17.89

7. A fair die is tossed once, what is the 11. Three men, Bedu, Bakare and koft
probability of obtaining neither 5 nor2? shared N500 in the ratio 3:2:x
a respectively. If Bedu‘s share is N150,
. find the value of x.
5 a. 1
/ b. 4
6 c. 5
b d. 6
.
2
/
3
C
.
I
d
.
1
/
6
e. 33/7

8. Find the mean of the number 1, 3, 4, 8, 8,


4, and 7
a. 4
b. 5
c. 6
d. 7
e. 8

9. A regular polygon has 9 sides. What is the


size of one its exterior angles?
a. 20°
b. 40°
c. 90°
d. 140°
e. 160°
e. 3 d. The disso of metallic sodium is
exothermic
12. The mean of a set of six numbers is
60. If the mean of first five is 50,
find the sixth number in the set
a. 105
b. 100
c. 95
d. 110
e. 75

13. A care dealer bought a second-hand car for


N250,000 and spent N70,000 refurbishing it. He
then sold the car for N400,000. What is the
percentage gain?
a. 60%
b. 32%
c. 25%
d. 20%
e. 35%

14. Find the number of sides of a regular polygon whose


interior angle is twice the exterior angle
a. 6
b. 2
C. 3
d. 8
e. 10

15. If the population of a town was 240,000 in January


1998 and it increased by 2% each year, what would
be the population of the town in January 2000?
a. 244,800
b. 249,600
c. 249,696
d. 480,000
e. 255,700

16. When sodium chloride and metallic sodium are each


dissolved in water:
a. Both processes are extol
b. Both processes are
endothermic
c. The dissolution of metallic
sodium is endothermic
e. The dissolution of sodium chloride is explosive d. Stands on one foot only
e. Stands on the toes of one foot
17. The periodic classification elements is
an arrangement elements in order of their:
a. Atomic weights
b. Isotopic weights
c. Molecular weights
d. Atomic numbers
e. Atomic masses

18. When heat is absorbed during a chemical


reaction, the reaction is said to be:
a. Thermodynamic
b. Exothermic
c. Isothermal
d. Endothermic
e. Thermostatic

19. Which of the following metals cannot


replace Hydrogen from water or from
steam?
a. Sodium
b. Magnesium Iron
d. Calcium
e. Copper

20. The boiling point of water is higher than that


of methanol because
a. Water is an oxide while methanol is
an alcohol
b. Inter-molecular forces in water are
stronger than those in methanol
c. Water is an inorganic compound
while methanol is
organic
d. Water is an ionic compound while methanol
is a covalent compound
e. Water is more reactive than methanol

21. A man will exert the


greatest pressure on a
bench when he:
a. Lies flat on his back
b. Lies flat on his belly
c. Stands on his feet
22. The clinical thermometer is day
characterized by having a: a. Speed
a. Wide range of temperature b. acidity
b. Wide bore c. Crate
c. Long stem d. Quickness
d. Construction e. Acceleration
e. Narrow bore

23. Which of the following instruments is 28. Our bus travelled at a high That
used to measure relative humidity?
a. Hydrometer
b. Barometer
c. Manometer
d. Hypsometer
e. Hygrometer

24. The ability of the eye to focus object


at different distances is called
a. Power
b. Accommodation
c. Normal vision
d. Persistence of vision
e. Long-sight

25. Which of the following particles or rays have the


greatest penetrating power?
a. Beta articles
b. Alpha particles
c. Gamma particles
d. Neutrons.
Complete each of the following sentences by choosing
the options that most suitably fill the spaces

26. Amos must keep His speed if he hopes


to succeed:
a. At
b. On
c. In
d. Up
e. On with
27. My mother the shell of the hard
boiled egg much quicker than I thought
a. Bust
b. Snapped,
c. Shattered
d. Cracked
e. Fractured
29. Jon‘s new house is very.... For the office
as e can get there is six minutes
a. Covenant
b. Available
c. Suitable
d. Pleasant
e. Comfortable

30. Most people can no longer


accommodate his long....
Story
a. Irritable
b. Boring
c. Weary
d. Tiring annoying

Each of questions of the next three questions, select


the option that best explains the information
conveyed in the sentence
31. The government warns that drink- driving is
punishable under the law:
a. Drinking and driving is
an offence
b. Driving while drinking is an offence
c. Driving while drunk is an offence
d. A drunkard driving can
be punished
e. Driving a drunk is an offence

32. The investigations stated clearly that they had


reached a dead end in their scrutiny of suspects
in the murder case
a. The investigators had evidence to sentence the
suspects to death in the end
b. The investigators did not know what to do with
suspects murder
c. The end had come for the suspects in the murder
case
d. There was no further progress in the
investigation of the murder suspects
e. The murder suspect had been pronounced dead. e. Wound

33. Everyone was ready to play the devils advocate in


the impeachment controversy
a. Everyone was willing to be an evil genius in the
controversy
b. Everyone was willing to the defenseless citizens the
consequences
c. Everyone was willing to unpopular point of view
the impeachment
d. Everyone was willing to speak against the
impeachment to encourage discussion on it
e. Everyone was willing to dine with the devil
In each of questions 34 to 38 choose the option opposite
in meaning to, the word or phrase in Italics
34. The ship was iniper/ledby high winds
a. destroyed
b. Saved
c. Piloted
d. deceived
e. Powered

35. Sailors are unusually dauntless in their exploits


a. Frightened
b. Selfless
c. Ruthless
d. Excited
e. Far sighted

36. The potency of the drug has been acknowledged


a. Inefficiency
b. Power
c. Loss
d. Action
e. Inavailability

37. The injured man is determined to


back at his assailant
a. Identify
b. Attacked
c. Forgive
d. Visit
38. The player writhed in pain after the
a. Walked out
b. Shook violently
c. Remained still
d. Cried out
e. Groaned

In each of questions choose the option completes the


gaps
39. After so many trials, the experiment
a. Paid off
b. Paid out
c. Paid for
d. Paid up
e. Paid in

40. The interpreter was wrong because he gave


the congregation a
a. Literacy
b. Literal
c. Lateral
d. Unilateral
e. Bilingual translation of the pastor‘s statement

41. The minister addressed the


workers to boost their
a. Morality
b. Moral
c. Morale
d. Mural
e. Extramural
42. All goes to the stadium regularly,
but he…………………..
a. Haven‘t been
b. Hadn‘t been
c. Hasn‘t been
d. Didn‘t go
e. Shall not be to the church for months.

43. Each producer is able to place a price on


his product by considering its
a. Choose
b. Assessment
c. Worth
d. Judgment
e. Shape
In each of questions 44 and 45, choose the option nearest
in meaning to the word(s) or phrase in italics
44. The party supporters vilified the chairman for e. ANPP
the role he played in the crisis that rocked the
party
a. Elected
b. Challenged
c. Condemned
d. Impeached
e. Praised

45. There was a glut of oil on the market.


a. A variety of
b. An accumulation of
c. An abundance of
d. An increase in
e. Scarcity

46. The capital of Jigawa State is a. bamaturu


b. Potiskum
c. butse
d. Kano
e. Abaji

47. The number of senatorial districts in


Nigeria are
a. 108
b. 109
c. 36
d. 54
e. 100

48. Edo State House of Assembly has


members
a. 18
b. 20
c. 22
d. 24
e. 30

49. Nigeria became a republic in the year


a. 1963
b. 1960
c. 1959
d. 1958
e. 1970

50. The Ondo State Governor was elected on the


platform of which party
a. PDP
b. AC
c. APGA
d. LP
51. The hold
a. Britain 57. Which of the following pairs of
b. Nigeria
c. South Africa
d. Angola

52. How many geopolitical zones are


there in Nigeria?
a. Britain
b. Nigeria
c. South Africa
d. Angola

52. How many geopolitical zones are there


in Nigeria?
a. 6
b. 8
c. 36
d. 9
e. 10

53. following states share their with their


names with their state capitals.
a. Anambra
b. Ogun
c. Jigawa
d. Gombe
e. Lagos

54. Honorable Minister of Health of


Nigeria is………………….
a. Ransome Kuti
b. Ayo Babalola
c. Babatunde Osotimehin
d. Godwin Abbe
e. Adenike Grange

55. The last military Head of


State Nigeria was
a. Gen. Abacha
b. Gen. Obasanjo
c. Gen. Gowon
d. Gen Baboginda
e. Gen. Abdusalami Abubakar

56. One characteristic of green plants


is that they
a. Possess specialized sense organs
b. Are usually motile
c. Are autotrophic
d. Respond slowly to stimuli
e. Are green
organisms are photosynthetic? 5. Ans: D
a. Amoeba and Paramecium They are social insects
b. Volvox and Rhizopus 6. Ans: B
7. Ans: B
c. Chalmdomoans and uglena Obtaining neither 5 nor 2 means obtaining either
d. Nostoc and Plasmodium 1,2,3 or 4.
e. Paramecium and tape worm
! ! ! !!2
58. In active transport, molecules move against = + + + /
! ! ! ! !
a concentration gradient, therefore
a. Molecules move from a region of low 8. Ans: B
!!!!!!!!!!!!! !"
concentration to a region of high concentration Mean = = =5
! !
b. Molecules move from a region of high 9. Ans: B
concentration to a region of low concentration Sum of exterior angles = 360
!"# !" !"#$%& !
Size of each angle =
c. The concentration gradient has little effect on the !" !"!!"#!"
movement of molecules !"#
d. Molecules force their way through special channels = = 40 !
!
e. No movement between compartments 10. Ans: B
!!!"!!"!!"!!"!!"!!"
Mean =
!
59. Raw materials required by green = 17.29(2d.p)
plants to manufacture their food are
a. Mainly fluid
11. Ans: C
b. Inorganic substances Total ratios = 3+2+x=5+x
c. Living materials Given that Bcdn‘s share = N150
d. Mainly gases !
500 = 150
e. Mainly minerals.
1500 = 150 (5+x) = 750 + 150
150 = 1500-750
60. Which of the following takes place when a X=750/150 = 5
person‘s temperature rises above 37°C
a. Sweating and vasoconstriction 12. Ans: D
Sum of 6 numbers -= x = 60 x 6 = 360
b. Panting and vasoconstriction
Ssum of 5 numbers = y = 50x5=250
c. Sweating and vasodilation Last number = x-y=360-250=110
d. Panting and vasodilation
e. Chills and vasoconstriction 13. Ans: C
Total cost of purchase =
250,000+70,000=320,000
ANSWERS TO 2010 NURSING QUESTIONS Profit/gain =
400,000-320,000=80,000
1. Ans: E
Such elements are called macro elements or
14. Ans: A
macronutrients
Sum of interior angles = 180(n-2)
2. Ans: D !"#(!!!)
For anacrobic respiration water is Size of an interior angle = !
!"#(!!!) !(!"#) !"#
replaced by ethanol (in plants) or lactic  =
acid (in animals). Saprophytes feed on dead decaying organic matter.
3. Ans: A
Taenia sagiriata has only suckers, no hooks
4. Ans: D
Xerophytes are found in dry places Halophytes
live in salty water Epiphytes are plants that grow
on other plants
! ! !
180(n-2) =
720 n-2 =
720/180 = 4 n
= 4+2 = 6

15. Ans: C
!
240000 = 4800
!""
New population in 1999
= 240,000+4800=244800
!
244800=4896 ―To reach a dead end‖ means ―there was no
!""
further progress in the investigation of the
New pop. In 2000 = 244800+4896=249,696 murder suspects
16. Ans: D
33. Ans: C
17. Ans: D
―To play the devil‘s advocate‖ means everyone
This is according to the modern periodic law as
was willing to defend an unpopular point of
stated by Moseley
view concerning the impeachment
18. Ans: D
Opposite in meaning
The opposite term is exothermic, where the
34. Ans: B
value of DH is negative
―Saved‖ opposite of ―imperiled‖ which means to
19. Ans: E
put something/somebody in danger
Copper is below hydrogen in the
electrochemical series
35. Ans: A
20. Ans: B
―Frightened‖ opposite of ―dauntless‖ which
Water contains stronger hydrogen bonds than
means not easily frightened or stopped from
mechanical or even ethanol.
doing something difficult
21. Ans: E 36. Ans: A
The smaller the surface area, the greater the ―Inefficiency‖ opposite of ―potency‖ which means the
pressure:
!"#$% power that somebody/thing has to affect you mind or
Pressure = body
!"#!

22. ---- 37. Ans. C


23. Ans: E ―Forgive‖ opposite of ―assailant‖ which means a
A hydrometer measures relative density person who attacks somebody, especially
physically
24. Ans: B 38. Ans. C
Accommodation is a function of the lens, which ―Remained still‖ opposite of ―writhed‖ which
it carries our by adjusting its diameter means to twist or move your body without
stopping, often because you are in great pain
25. Ans; D 39. Ans. D
―Keep up‖ her means to make something stay at a ―Paid up‖ means to strongly support
high level‖ somebody/thing
40. Ans. E
27. Ans: D Bilingual translation of the pastor‘s statement
―Cracked‖ means ―to break open or into 41. Ans. C
piece‖ ―Morale‖ means ―confidence and enthusiasm‖
42. Ans. C
28. Ans: A ―Hansn‘t been‖ used to denote action that has
―Speed‖ means ―the rate at which somebody or not be completed
thing moves or travels. 43. Ans. C
29. Ans: C ―Worth‖ means ―values, importance‖ nearest in
―Suitable‖ means ―in a way that is right or meaning (synonyms)
appropriate for a particular purpose or occasion 44. Ans. C
―Condemned‖ same as ―vilified‖ which means
30. Ans: B to say or write unpleasant thing about
―Boring‖ means uninteresting and tiring somebody/thing
Best explained information 45. Ans. C
31. Ans: D ―Glut of oil‖ means as ―abundance of oil‖
―a drunkard driving can be punished‖ same as drink 46. Ans. C Damaturu is the capital of Yobe Kano is
driving‖ the capital of Kano state
The others are not state capitals.
32. Ans: D 47. Ans. B
36 states with 3 members each gives a total of 108 your answer booklet
members in the 108 senatorial district. The last one Johnson is a stubborn man, he will
is from the FCT Abuja. never His words
48. Ans. A
Each represents a local government area of the state.
49. Ans. A
That was after independence in 1960
50. Ans. D
51. Ans. C
2018 world cup will in hold Russia
52. Ans. A
They are:
North central
North-east
North-west
South-west
South-east and
South-south
53. Ans. D
Others are Kano, Kaduna and katsina
54. Ans. C
The current health minister is prof. Isaac Adewole
55. Ans. E
Abdulsalam Alhaji Abubakar (Rtd) left office in
1999
56. Ans. C
This means that they can manufacture their food
57. Ans. C
They both posses chlorophyll
58. Ans. A
Active transport usually requires energy but passive
transport does not
59. Ans. B
Such inorganic substances include CO2 and H2O
60. Ans. C
370c is regarded as the normal body temperature. At
temperature greater than this, there is vasodilatation to
facilitate heat losss.

2008
ENTRANCE EXAMINATION TO SCHOOL
OF NURSING UNIVERSITY OF BENIN
TEACHING HOSPITAL, USELU, BENIN
CITY GENERAL NURSING PAST
QUESTIONS AND ANSWERS
INSTRUCTION TO ALL CANDIDATES
1. Your examination number (Not Name) must be
write on every page of your question paper which is
a. Chew d. Grab
b. Spit CHOOSE THE OPTION A-b THAT IS
c. Eat
d. Bite
2. Kindly me your book because my friend has mine
a. Borrow/ borrowed
b. Borrow/lent
c. Lend/lent
d. Lend/borrowed
3. Two young boys have been caught with ports of
the stolen machine but Admitted, stealing it
a. Neither of them has
b. Neither of them have
c. None of them has
e. None of them have
4. bouglas is now a Student
but it took him years to
a. Matured/matured
b. Mature/mature
c. Mature/mature
d. Matured/ maturez

5. The rebels will soon come back.


We have been informed their
a. Of/predicament
b. About/indulge
c. On/rearmament
d. As for/advancement
6. The adventures rain into
many In the forest
a. bear
b. bears
c. beers
d. beer
7. The Nurses class More of
girls than boys this session
a. Comprised of
b. Comprise of
c. Comprise
d. Comprises
8. I shall work hard so that I can a distinction in
English, mathematics, biology/Health science,
chemistry and physics
a. Secure
b. Make
c. Receive
NEAREST TO MEANING TO THE Words b. Those who displayed the notice
and perhaps writer then
OR PHASE?
BELOW (9-12)
I was told that it is an extinct volcano Knowledge,
having been wallowing in the darkness of ignore
even then I was not be able to escape the thought
that though I would not be sure how much of the
responsibility for the darkness was mine, the fact
that something had not been done to draw attention
to that apocalypse of notice board had contributed
clearly is my initial predicament. All that
notwithstanding, I
learnt from that incident an indelible lesson which was
to stand me in very good stead though my stay in the
university, namely, the need to be vigilant and
observant.
9. The writer of the passage felt isolated because
a. There were too many people
b. He was a new student. in Uni ben
c. There were too many new students
d. None of the many students around could help
him.

10. The writer say‘s that the prospect not cheerful


because
a. No one could tell him how to
get to Tunubu tall
b. He had felt too elated when he gained admission
with the university
c. There was no hope of getting out if his
predicament
d. His other students were hostile

11. In the passage, the darkness of ignorance ―refers to


the
a. State of the writer before he found the notice
board
b. Ignore of illiterates
c. Ignorance of those who have not been to Uniben
d. Fact that he was a freshman

12. According to the passage, the writer‘s predicament


was caused by
a. The writer himself
c. The nature of the notice c. Resistant
board and where it was placed d. Gas-liquid
d. Those who displayed the notices 19. During which era, did Nigeria U- 17
boys won the world cup final/year
13. The writer remembered his first day in
a. Samson Secia/2007
the university for a long time because
he. b. Augustine Evavon/2007
a. Had felt very happy at being admitted into
the university
b. Could not get anybody to help him on his first
day in the university
c. Had learnt the hard way how to keep
informed about happenings in the university
d. Eventually discovered the notice board by chance

14. The Chief Medical Director of UBTH


a. Prof. E.A C Nwanzi
b. Prof. J.P Okpede
c. Prof. F. Okonufua
d. Dr. M. Ibadiri

15. The commissioner of health Edo state.


a. Prof. N.P Okpere
b. Dr. Moses Ibadin
c. Dr. Moses Momoh
e. Dr. (Med) b.O. Qkoh Motto for SCHOOL OF
NURSING UNIVERSITY OF BENIN
TEACHING HOSPITAL, USELU, BENIN
CITY GENERAL

16. the motto for SCHOOL OF NURSING


UNIVERSITY OF BENIN TEACHING
HOSPITAL, USELU, BENIN CITY been
a. Knowledge is power
b. Knowledge to serve
c. Knowledge to serve humanity
d. Knowledge for service
17. A Serpent that is raped around a pole of
150 inches tall. In the
hospitals represents
a. The traditional emblem of the
medical profession
b. The traditional emblem of the
nursing profession
c. The traditional emblem of the nursing
and medical profession
d. The healing of the nation
18. Which of these thermometers,
nurses used for temperature reading
a. Mercury glass
b. Clinical
c. Yemi Teller/2007
d. Bulfe Joe/2007 c. N.P.I - National Programme of immunization
20. What year/month did the new
currency notes and coin where
reissued with a new design.
a. 2004120TH of February
b. 2005/28TH of February
c. 2006/28th of February
d. 2007/28TH of February
21. Which of these options below is true
of the seat of government?
a. Calabar - Lokoja - Lagos -
Abuja
b. Calabar - Lagos - Lokoja -
Abuja
c. Lokoja - Calabar - Lagos -
Abuja
d. Lagos - Lokoja - Calabar -
A b uja
22. Which of these names and office of operation those
not correspond to each other
a. Chief Ken Nnamani/Senate President
b. Hon. Justice Salute moddibo Alfa Belgore, CON,
GCON/Chief Justice of Nigeria
c. Sunday Ehindero/Nigeria police leader
d. Prof. Grace Alele Williams/the l female vice-
chancellor of uniben
23. The following abbreviation/meaning not correspond
a. N.A.N.N.M. - Nigeria Medical Association
b. NbbC - Niger Delta development Commission
c. S.O.N - School of Nurse
d. N.I.S.ON.M.G - Nigeria Schools of nursing
midwifery games
24.
a. N.A.N.N.M National Association of Nigeria Nurses
and Midwives
b. N.A.S.N.M.N - National
Association of Student
Nurses and Midwives of
Nigeria
c. N.M.C.N - Nursing and Midwifery Council of
Nigeria
d. W.A.C.N - West African College of Nursing

25.
a. M.B.C.N - Medical and dental Council of Nigeria
b. N.H.I.S - National Health Insurance Scheme
d. SAVAN - Save Accident Victim Association 33. Calculate the molar mass of the following
of Nigeria CusO4.5H20. (Hint:Cu 63.5, 5:32, 0,: 16)

26. The prevention arid control of major diseases in


the world is one of the functions of the
a. FAQ
b. WHO
c. Red Cross Society
d. UNESCO

27. buring development the transformation that


occurs from one stage to the other is called
a. Embryology
b. Metamorphosis
c. dimorphism
d. None of the above

28. The parasite that causes sleeping sickness is


a. Tse tse fly
b. Mosquito
c. Aristole
d. Gregor Mendel
e. Charles barwin

29. The study of heredity started with


a. convenient
b. Pallon
c. Trypanosoma
d. Trichuny

30. Which of the


following linked trait
a. Colour blindness
b. Haemophilia
c. Baldness
31. d. beafness
31. Enzymes that break up are called
a. Amylase
b. Triglycerides
c. Lipases
d. Proteases

32. What are number of Neutrons and


electrons following species.
a. p(5)N(4)&E(5)
b. p(9), N(5) & E (14)
c. p(14)N(4)&E(9)
d. p(4)N(5)&E(9)
a. 149.5g
b. 249.5g 39
c. 349.5g
d. 49.5g

34. 15.0cm of limus paper (blue) was dip into, a


highly concentrated. H25o4 What will be the
colour change
a. bark blue
b. Red Colour
c. Purple
d. Violent

PHYSICS
35. What quantity of copper will be deposited by the
same quantity of electricity of that deposited l8g of
aluminum [A] 27, cu 64
a. 64g
b. 32g
c. 16g
d.

36. A step-up transformer is designed to operate from


25v supply. If the transformer is 80% efficient,
determine the current in the primary coil when the
output terminals are connected to 24v. 100w lamp.

a. 5.OA
b. 4.OA
C. 2.5A
d. 2.OA

37. Which of the following pair consist of fundamental


quantities only
a. Velocity and gravitational potential
b. Acceleration
c. Momentum & work done
d. Moment mass

38. One of the limitations of Thompsons model of the


atom is that it does not explain
a. Small angle scattering
b. Stability of the atom
c. Ionization process
d. The variation of the effective atomic radius
An atom is said to be excited if an electron of the
6. Ans. D
atom is
―Deer‖ because zero plural do not take the plural markers
a. In the ground state ‗s‘. ‗es‘ etc
b. At infinity 7. Ans. D
c. Promoted to air energy level higher than ―comprises‖ because it does not collocate with ―of‖
its original level 8. Ans. B
d. Having an energy value of 0.0ev.

40. Express P in terms of other letters if p+x


= 1 P+y q
a.
b. Y

qx
QP
—P
q+1
y—
qx
c. 2i
zE y
- qx
d. q

1

ANSWERS TO 2008 NURSING


QUESTIONS
1. Ans. C
―To eat ones word‖ means ―to apologize‖ or ―to
admit that what you said was wrong‖

2. Ans. C
―Lend/borrowed‖ because ―borrow‖ means to take
or receive while ―lend‖ means ―to give‖
3. Ans. C
―neither of then have‖ because neither differs from
―no‖, ―none‖ neither is used only with a set of two
person or things, while ―none‖ is used with three or
more entities‖
4. Ans. B
―mature/mature‖ as an adjective it does not take
―ed‖ or ‗s‘ for, but verb the past and pas participle is
―matured‖

5. Ans. B
―informed about their indulge‖ because ―informed‖
collocates with ―of/about‖
―Make‖ means to achieve something e.g
―distinction‖ 29. Ans. C
9. Ans. B
He was obviously a new student Mendel is the father of the Heredity.
10. Ans. C Darwins work was on evolution
There was no hope of getting out of his predicament
11. Ans. D
30. Ans. D
―fact that he was a freshman‖
Sex-linked traits are controlled by sex-linked
12. Ans. C genes, which are located on the sex
―Thee nature of the notice board and where it was chromossomes
placed‖

13. Ans. C 31. Ans. A


Had learnt the hard way how to keep informed about Precisely, amylases break up Amylon (Starch)
happening in the university

14. Ans. D 32. Ans. A


Prof. M. O. Ibadin was CMD of UBTH for P=5
eight(8) years and only left office in 2017 E = 5 (same as P)
N=9–5=4
15. Ans. C
Professor Moses monoh is a Preofessor and
33. Ans. B
Consultant genral Surgeon in the UBTH
CuSO4 5H2) + Cu + S + 40 +5H2O
16. Ans. C = 63.5 + 32 + 64 + 90
17. Ans. A = 249.5g
The emblem is called ―CADUCEUS‖
18. Ans. B 34. Ans. B
19. Ans: None of the options is correct Acids turn blue litmus red while alkalis turn red
20. Ans. D litmus blue.
21. Ans. A
Abuja, which is located at the centre of Nigeria, is
the current capital city of Nigeria 35. Ans. A
!! !! !! ! !
22. Ans: None of the options is correct !"!! !"!!"
M1 = 18 C1 = 3 RAM1 = 27
23. Ans. Only B is actually correct! M2 = X 18 C2 = 2 RAM2 = 64
!! ! !"# !" ! ! ! !"
! !
24. Ans: All of the above M= = 64g
! !"#! ! ! !"
!
25. Ans: All of the above
26. Ans. B 36. Ans. B
The WHO (World Health Organization) is 35, Sleeping sickness (Trypanosomiasis) is called by
one of the arms of the United Nations (UN) Trypanossoma brucci gamniense

27. Ans. B
Metamorphosis may be complete or incomplete.

28. Ans. C
37. Ans: None of the options is
correct 38: Ans.
A
39. Ans. C

40. Ans. D
!
=
!

Cross
multiply q (p
+ x) = p + y
Pq + qx = p +
y
Pq - p = y – qy Choose the word or phrase form a c% e which has same
P(q-1) = y – qx meaning as the underlined word(s) in each sentence

Divide both sides by (q-1)


5. He has been a good mediators even in
!(!!! (!!!"
= minor family disputes because the
(!!!) (!!!)
!!!"#(!!! cases his view are always aundicated
P= a. Coloured
(!!!)
2007 b. Sick
c. Suspicious
ENTRANCE EXAMINATION TO SCHOOL
d. Unscrupulous
OF NURSING UNIVERSITY OF BENIN
e. Prejudiced
TEACHING HOSPITAL, USELU, BENIN
CITY GENERAL NURSING PAST
6. When a man is immune to a illness. He is
QUESTIONS AND ANSWERS
a. Oppose to attended to it
b. Hated by it
1. Obasanjo claims to be fighting
-- goal for Nigerians c. Protected by it
d. Addicted to it
a. Curse
b. Course
7. Our school prefect and we hate him behavior
c. Coase
a. Efficient
d. Case
b. Efficacious
c. Over jealous
2. She doesn‘t have to attend the
d. Active
lecture
e. Show
a. Doesn‘t she
b. Is she
8. One of the surest ways to ensure good health is to
c. Has she
have a wholesome and adequate diet
d. Does she
a. Palatable
e. Isn‘t she
b. Mixed
3. Many of those who………………. the country c. Hygiene
d. Delicious
educational policy have taught for many years
d. Health giving
a. Develop
b. Fabricate
c. Organize
d. Formulate
Choose from the words letter a to A that is nearly
opposite in meaning to the underline words
4. After our series of quarrel it would
9. Buhari‘s mother was crossed with him
be to pretend that I have come more regard for him
for breaking their clay pot
a. Hypothetical
b. Essential a. Annoyed with
b. Naughty with
c. Hypocritical c. Frank with
d. Necessary d. Happy with
e. Inventable e. Irritated

10. Every teacher likes an obedient


student
a. Intelligent
b. Lazy
c. Active cylindrical drum 28cm in diameter and 50cm deep
d. Closest hold?
e. Factual

11. Musa is always kite to school


a. The last
b. Punctual
c. The first
d. Regular
e. Factual

Tick the most appropriate option nearest in meaning to


the underlined word
12. Femi is much in request today
a. Making much request
b. Asking rather much
c. Commonly asked for
d. Ready to be used for request
e. Robust

13. Oba is a ferocious young man; he is


rather difficult to associate with
a. Violent
b. Too good
c. Lazy
d. Stingy
e. decent

14. Adam is so slow that he cannot make a good score


in football
a. Clever
b. bull
c. Sensible
d. Sluggish
e. Blind

MATHEMATICS
15. Evaluate 6.2972x0.251. Correct to three decimal
place
a. 158.039
b. 15.801
c. 1.600
d. 1.581
e. 158

16. How many liters of oil does a


a. 308liters form a. 145 x10-4
b. 308c b. 145 x10-3
m2 c. c. 145 x 10-1
1600 d. 145 x10-2
d. 1581
e. 158

18. Seven boys scored the


following marks in their entrance
exams
340,420,570,250,399,410,520 the
mean mark is
a. 414.3
b. 413
c. 541.3
d. 441
e. 531.4
c.
d.
17. Solve the following equation 2(2) + 7 (168-0) 0
a. x-3
b. x-2
c. x6
d. x -5
e. x=5
18. Seven boys scored the following
marks in their entrance exams
340,420,570,250,399,410,520 the mean is mark
a. 414.3
b. 413
c. 541.3
d. 441
e. 531.4
19. 5implify 15 plus 17 plus 19 plus 21
a. 75
b. 76
c. 72
d. 70
e. 82
20. Expand the following (a+2b) (o-
2b) a. 1-4b2
b. a2 46
c. a4b
d. a2 + 4b2
e. a2 + 4b2
21. Expand 000145 in standard
!!! d. Endoplasm
22. Simplify
!!! e. Nuclei
a. x-y
b. x2+y2
c. (x-y)2
d. 2x+4y
23. Which of the following statement is
not true?
a 18 5
b. 30.45
c. 0.2
d. 0 5
e -2>2
24. In an examination 35 out of 125
students failed, what percentage
passed.
a. 28%
b. 35%
C. 65%
d. 72%
e. 90%
25. Human reproductive organ include? (i)
Ovary (ii) Spermatozoa (iii) breast (iv)
testis (v) uterus
a. i, ii and iii only
b. I,iv and v only
c. ii, ii, iv, v only
d. ii v only
e. I, iii, iv, v only
26. A meal of fried eggs, yam and fruit
salad contains only
a. glycogen, cellulose, vitamins
maltose, and water
b. Cellulose, protein, mineral
salts, fats and vitamins
c. Protein fats mineral salt
vitamin water
d. Vitamin mineral salt protein
fats carbohydrates and water
e. Oil, carbohydrates and
vitamins
27. The structure for movement in
amoeba?
a. Ectoplasm
b. Pseudopodium
c. Flagellum
the
28. The following levels
a. Blood plasma and the red blood cells are isotonic
of organization in living
b. Red blood cells are hypotonic to the plasma
things in the correct
c. The red blood cells are hypertonic to the plasma
sequence starting from
d. The red blood cells receive digested nutrient
the simplest to the
e. None of the above
most complex
32. Micro-organisms are important in the nutrient cycle
a. Tissue - Cell - Organ -system
because
b. System—organ—tissue-cell
a. They are capable of multiplying quickly
C. Cell - tissue - system - organ
b. They are eaten by large organism
d. Cell-tissue-organ-system
c. They break down dead organic materials
29. The ability of the eye to alter the
d. Many of them are parasites
focal length of the lens with regards to
e. All of the above
the distances of the object from the eye
33. In palm wine, yeast acts as a?
is known as? a. Carrier
a. Binocular b. Culturing agent
b. Nocturnal vision c. Preservative
c. Distortion d. Catalyst
e. Sweetening agent
d. Accommodation
34. Which of the following is more
e. Focusing concentrated in the blood within the
30. During blood transfusion
agglutination may occur as a result of
the reaction
between,
a. Si
milar
antige
ns
and
antib
odies
b. Contra
sting
antigen
and
antibodie
s
c. Two different antibodies
d. White
bloods
cells
from
individua
ls
31. A major reason why the blood
cells can survive in the blood is that
renal artery than in the blood within a. Antibiotics
the renal vein? b. Antibodies
a. Urea
b. Red blood
C. Glucose
d. Protein
e. White blood cells
35. The sex of a baby is determined by the? a. Vaginal
receptivity
b. Cell type
c. Egg size
d. Sperm agility
e. Male strength
36. The plants that can survive in
extreme dry places are called?
a. Epiphytes
b. Halophytes
d. Neophytes
e. Xerophytes
37. The form of nutrient mainly stored in
the liver is?
a. Fructose
b. Fatty acid
c. Glucose
d. Amino acid
e. Glycogen
38. The main function of the respiratory
tract is?
a. Exchange ofO
b. Exchange of CO2
c. Exchange of CO
d. Exchange of SO2
e. a and b
39. Which of the following equations best represents
photosynthesis?
a. CO2 + H20 + energy — starch + chlorophyll
b. CO2 + Chlorophyll + energy— starch + oxygen
c. CO2 + energy —‗Glucose + 02
d. Glucose+Co2 —‗starch + Oxygen
e. Glucose + 02—‘ Co2 + H20 + Energy
40. The part of the brain that controls
involuntary actions is in the?
a. Cerebellum
b. Cerebrum
c. Hypothalamus
d. Medulla Oblongata
e. Thalamus
41. When microbes invade the body they
produce poisonous substances called
c. Toxins soap easily
d. Vaccines d. It contain too much oxygen
e. Antigens
42, Which of the following is not
viviparous?
a. bog
b. Pigeon
c. Sheep
d. Goat
e. Cow
43. The ability of organism to maintain a
constant internal environment is known
a. Diuresis
b. Endosmosis
c. Homeostasis
e. Plasmolysis
44. The Bicuspid valve is located between
a. Left atrium and left ventricle
b. Aorta and ventricle
c. Superior venacava and
d. Right atrium and
right ventricle
45. The brain and the spinal cord make up
a. The peripheral nervous system
b. Autonomic nervous system
c. Central nervous system
d. Sympathetic nervous
46. The vector of the malaria
parasite is the?
a. Male anopheles
b. Bacteria
c. Nematodes
d. Nemotodes
e. Cercariae
47. The mixture contains………….
a. Two or more constituents
b. One component
c. Three or four component
d. None of the above
e. All the above
48. Water is said to be hard when……..
a. It contains too much hydrogen
b. It does not form later
readily with soap
c. It forms lather with
e. It contain less oxygen c. Remain constant
d. Be zero
49. A compound found to contains 40%
carbon. 53.3% oxygen and 6.7%
hydrogen it empirical formula is?
a. CH402
b. C2H4
C. CH2O
d. CH4O
e. C2H402

50. The concentration of oxygen in room air


a. 78%
b. 0.03%
c. 12%
d. 21.5%
e. 21%
51. Which of the following establishment use the
process of fermentation in its operations?
a. Textile industries
b. Brewery
c. Block 1ndustry
d. Soap manufacturing industry
e. Fertilizer plant
52. Which of the features of a human skeleton can be
determined in carbon- dating?
a. Height
b. Mass
c. Age
d. Weight
53. Calculate the percentage by mass of sodium in
anhydrous sodium tetraoxosulphate Na2504 (Na + 23, 5
32) =16
a. 50%
b. 30.4%
c. 32.4%
d. 40.3%
e. 20%
54. Which of the following is a measure of the average
kinetic energy of the molecules of a substance?
a. Mass
b. Pressure
c. Temperature
d. Weight
e. All of the above
55. Carbohydrates are compounds
containing carbon. Hydrogen and oxygen in the ratio of?
a. 3:1:1
b. 2:1:1
c. 4:1:1
d. 1:2:1
56. According to the kinetic theory increase in
temperature causes the kinetic energy of particles to
a. Decrease
b. Increase
e. Be parallel burning in air contributes to acid rain?
57. A sample of orange Juice is suspected to have a. Sulphur
been contaminated with a yellow dye which of the b. Magnesium
follow methods can be used to detect the dye?
a. Decantation
b. Chromatography
c. Distillation
d. Filtration
e. Evaporation
58. Which of the following
pollutant is biodegradable?
a. Sewage
b. Plastics
c. Metal scups
d. Lead compound
e. Hydrogen sulphide
59. Nuclear reaction can be used in the following except
a. Gauging the thickness of objects
b. Making atomic bombs
c. Curing cancer
d. Generating electricity
e. Purifying
60. Petrol can be obtained from diesel by?
a. Distillation
b. Cracking
c. Catalysis
d. Polymerization
e. Dehydrogenation.
61. Electrovalent compounds
are characterized by
a. Solubility in ethanol
b. High molar mass
c. High melting point
d. Strong oxidizing ability
e. Catalytic property
62. The type of chemical bond that exists
between potassium and oxygen in potassium
oxide is?
a. Ionic
b. Metallic
c. Covalent
d. Dative
e. Dipole-dipole attraction
63. A radioactive solid is best stored by?
a. Under paraffin oil
b. Under ultraviolet light
c. In a cool dark cupboard
d. Between thin sheets of metal

64. Which of the following on


c. Aluminum b. Temperature
d. Zinc c. Moss
e. Iron
65. Sieving is a technique used to separate mixtures
containing solid particles of
a. Small sizes,
b. Large sizes
c. Different size
d. Small and large sizes
e. Extra large sizes
66. Environment pollution is worsened by the release
from automobile exhausts of
a. Hydrochloric
b. Water vapor
c. Steam
d. Smoke
e. Heavy metals
67. In the reaction above HCL(aq)+ H20 H3O+ (aq) + C1-
(aq) CL- (aq) is the
a. Conjugate acid
b. Acid
c. Alkaline
d. Base
e. Conjugate base
68. Power is defined‘ as the?
a. Velocity to extent force
b. Product of force and time
c. Product of force and distance
d. Ability to work
e. Energy expended per unit time
69. Power is defined as the?
a. Capacity to extent force
b. Product of force and time
c. Product of force and distance
d. Ability to work
e. Energy expended per unit time
70. A body moves with constant speed but
has an acceleration the body is?
a. Moves in striate line
b. Moves in circle
c. Is oscillating
d. Is in equilibrium
e. Has a varying acceleration
71. As orange fruit drops to the ground from top of a tree
45cm tall. How long does it take to reach the ground? G
lOms4
a. 3.Os
b. 4.5s
c. 6.02
d. 7.5s
e. 9.Os
72. The saturation vapor pressure of a
liquid depends on its?
a. Volume
d. Density d. 12.252
e. Pressure e. 806.752
73. Which of the following has 79. The pressure exerted by a liquid is dependent on the?
the highest surface tension? a. Mass of the liquid
a. Cold water b. Cross sectional area of the container
b. Soapy water c. Density of the liquid
c. Warm water d. All of the above
d. Oily water
e. Salty water
74. Which of the following are emitted from a
radioactive substance without altering either the
nucleon number or the proton number of substance?
a. Gramma rays
b. Alpha particles
c. Beta particles
d. Protons
d. Neutrons
75. An object weights 10.N in air and
7.0N in water. What is its weights when immersed
in liquid of relative density 1.5?
a.
4.50N
b.
4.67N
c.
5.57N
d. None of the above
76. A plane incline at an angle of 3Q0 to the
horizontal has an efficient of 50% The force parallel to
the plane required to push a load to 10N uniformly up
the plane is?
a.
40.3N
b.
2.00
c. 1.33
d. 150
e. 0.25

77. A concave mirror of radius of


curvature 20cm has a pin placed at
15cm from the pole. What will be
the magnification of the image
formed? a. (4.00)
b. 2.00
c. 1.33
d. 1.50
e. 0.25
78. An electric bulb is rated 60.w 220v.
Calculate the resistance of
its element when it is
operating? ci. 296.752
b. 400.025
c. 375.052
80. The running mate for President b. Delta, Ondo, Kogi & Ekiti
Olusegun Obasanjo 1999 election was? c. Delta, Oyo, Ekiti & Kogi
a. Dr. Olushola saraki d. Imo, Ondo, Lagos & Ekiti
b. Asiwaju Bola Ahmed Tinubu e. Aba, Lagos, Kogi & Lokoja
c. Atiku Abubakar 89. The name of the current pope is ?
d. Allaje Umaru Diku a. Emeka Arienze
e. Gen. Abolsalam Abubakar b. Josef Razinger
81. The current chief medical director of C. Kim Basinger
UBTH is? d. Carlif Wortyla
a. Prof. CE Nwane 90. The director general of the Nigeria
b. Prof J.E Okpade stock exchange is
c. Prof Eyintayo Lambo a. Dr. Dora Akunjili
d. Prof AU Oronsaye b. Dr Ndi Okereke Onyuike
e. Akenzua c. Carlif Wortyla
82. Edo state was part of? d. George paton
a. Western region 91. Who is the world footballer of the year?
b. Midwest region a. Ronaldo
c. None of the above b. bavid Beckam
d. All the above c. Jay Jay Okocha
d. Ronaldinho
83. The tsunami disaster occurred on? 92. The ―confluence state‖ is
a. 26th December 2004 a. Niger
b. 25th December 2004 b. Kogi
c. 25th May, 2005 C. Benue
d. 24th December 2004 d. Delta
84. The secretary General of United 93. Who is the first female justice of the
Nations is? supreme court of Nigeria?
a. Sule Gambari a. Prof. Bokunle Awe
b. Kofi Anna b. Hon. Justice Atinuke Ige
c. Tom Ikimi c. Hon Justice Fumilayo OniOkpaku
d. Adudu Ogbeh d. Hon. Justice Alom Mariam Mukhkur
e. Don Etiebet e. Hon Esohe Jacob
85. A Nigeria Government programme to fight 94.. The full meaning of OPEC is
corruption is a. Organization of petroleum exporting countries
a. INEC b. Officer of petroleum exploiting countries
b. NCC c. Organization of petroleum extracting communities
c. NAFD d. Organization peace enforcement Cedet
d. EFCC e. Offershore petroleum
e. NEED extracting company.
86. Which of the following is an example
of a computer hardware?
a. CPU box
b. Keyboard
c. Monitor
d. Mouse
e. All the bove ANSWER TO 2007 UNRSING PAST-QUESTION
87. The following are routes of
contracting HIV Aids except? 1. Ans: B
a. Shaking hands a. Anambra, Kono, Enugu & Ondo
b. Mother to child
c. Sexually intercourse
d. Needles
e. Blood transfusion
88. The states that share boundary with
Edo States are?
“ Course” meaning ―course of
action‖ a way of acting in or
dealing with a particular situation
2. Ans: D
“Does she” because a positive
statement goes with a negative tag
3. Ans: D Vol. = 308000cm3
Vol = 08dm3 (08litles)
“Formulate” means to create or prepare something
carefully, giving particular attention to the details. 17. Ans: No correct option
2(2) + 7(4x) – (168 - 0) = 0
Policy/theory/plan/proposal are all part for formulation
4 + 28x – 168 = 0
4. Ans: C 28x – 164 = 0
28x = 0 + 164 = 164
“Hypocritical” which means to pretend to have moral !"#
X=
standards or opinions !"
18. Ans: No correct option!
5. Ans: E
“Prejudiced” which means having an unreasonable Mean
dislike of or preference for somebody or thing especially !!"!!"#!!"#!!"#!!!!!"#!!"#
based on race, religion, sex etc some as ―jaundicated, =
!
bias‖ = 415.57
6. Ans: C
“Immune” same as ―protected by it‖
19. Ans: C
7. Ans: C 15 plus 17 plus 19 plus 21
“Over zealous”, self-important‖ same as ―officious‖ = 15 +17 +19 + 21
= 72
which means too ready to tell people what to do or to use
power you have to give orders 20. Ans: E
(a+2b) (a-2b)
8. Ans: E
A2 – 2ab + 2ab – 4b2
“Health giving” same as ―a wholesome and adequate‖
= a2 – 4b2
which means good fir your health opposite in meaning
9. Ans: E
21. Ans: B
“To cross with somebody” means to be annoyed with
him/her the opposite is ―happy with‖ 22. Ans: -

10. Ans: B 23. Ans: -


“Lazy” means unwilling to work or be active‖ the
opposite is ―obedient‖ 24. Ans: D
No of students that passed = 25 – 5 = 90
11. Ans: B !"
Percentage pass = x 100 = 72%
“Punctual” means happening or doing something at the !"#
arranged or correct time; not late the opposite is ―late‖
Nearest in meaning 25. Ans: B
26. Ans: D
12. Ans: C
“commonly asked for” same as ―much in request‖ 27. Ans: B
Flagellum is present in Euglena
13. Ans: A 28. Ans: D
“violent” same as ―ferocious‖ meaning very aggressive or
violent 29. Ans: D
This is made possible by the culinary muscles
14. Ans: D
“sluggish” same as ―so slow‖ 30. Ans: A

15. Ans: D 31. Ans: A


16. Ans: A Isotomic means having the same
!! !! !"!!"
Vol = h= x x 50 concentration
! ! !
32. Ans: C
33. Ans: C Beers are made in Breweries. The soap manufacturing
Yeast produces enzymes that act as catalyst
industry utilizes the saponfication process.
34. Ans: A
Impurities are more concentrated in the Renal artery
35. Ans: - 52. Ans: C
36. Ans: D
Epiphytes are plants that grow on the trees Carbon-dating is hinged on measuring the carbon 14
content of bone
37. Ans: E
glycogen
38. Ans: E 53. Ans: C
Na2So4 = 2N2 + S + 4O
39. Ans: B = 2(23) + 32 + 4(16)
40. Ans: D = 46 + 32 + 64 = 142
!"
% Na = x 100 = 32.4
The cerebrum controls VOLUNTARY Actions !"#
41. Ans: C
54. Ans: C
Antibodies are used by the body to fight theses taxins This is one of the postulates of the kinetic energy
42. Ans: B
55. Ans: D
The pigeon is oviparous However, this is only truth for Monosaccharide but not
43. Ans: C for Disaccharide or polysaccharides
56. Ans: B
44. Ans: A
The particles acquire energy and move faster
The Bicuspid valve is also called MITRAL VALVE . the
tricuspid valve is on the Right side 57. Ans: B
45. Ans: C
Chromatography is especially suitable for the separation
46. Ans: -
of coloured substances
47. Ans: A
58. Ans: A
These components are not chemically Bonded together.
Many synthetic compounds are Non-biodegradable
48. Ans: B
This is due to its containing calcium and for magnesium 59. Ans: D
ions.
Nuclear reaction generate nuclear Energy which is a
49. Ans: C
C O H separate form of Energy from electrical energy.
!" !".! !.! However, as usual energy can be converted from one
!" !" !
form to another
3.33 3.33 6.7
60. Ans: B
Divide through by the smallest value
Cracking breaks longer hydrocarbon claims into shorter
!.!! !.!! !.!
!.!! !.!! !.!! ones
1 1 1 61. Ans: C
Empirical formula = CH2O They are very soluble in water and are mostly solids at
room temperature
50. Ans: E
62. Ans: A

63. Ans: D

51. Ans: B 64. Ans: A


It forms non-
metallic oxides
which are acidic
in solution. The
other form basic
oxides in solution
65. Ans: C Magnification = = 30 15 = 2.00
66. Ans: E
78. Ans: E
Heavy metals can cause serious poisoning. Affecting the V = IR; I =
brain and kidneys. Examples of heavy metals are lead P = IV !
!
and mercury ∴ P=
67. Ans: E ! !
!!"
The conjugate base of an acid differs from the acid by 602 = = 306.67
!"
just one proton.
68. Ans: E 79. Ans: D
Mathematically:
!"#$% × !"#$%&'( !"#$%&
Power = =
!"#$ !"#$ 80. Ans: C
69. Ans: E He is known as the ―Turakin Adamawa‖
See 68 above
70. Ans: B 81. Ans: D
71. Ans: A 82. Ans: B
As it drops from rest, u = o
! Then it was later known as Bendel state before
∴ S = gt2
!
becoming Edo state in 1991
45 = (10) (t)2
!
83. Ans: A
5t2 = 45
!"
T2 = = 9
! 84. Ans: B
t = 9 = 3.05 As of the time of this exam, Kofi Annan was secretary
General.
72. Ans: B
85. Ans: D
73. Ans: D EFCCC means ―Economic and Financial Crimes
74. Ans: A Commission‖

They do not have mass or charge 86. Ans: E

87. Ans: A
75. Ans: D Eating from the same plate and kissing are not
Let this weight in liquid be x recognized routes of HIV Transmission
!"!!
Then = 15
88.

89.

90.

91.
f= 2 = 202 = 10cm
The record is currently held by Christiano Ronaldo of
u = 15 Real Madrid
using the mirror formula 92. Ans: B
! ! !
= +
!
!
!
!
!
!
93. Ans: D
!" !" !
!!! 94. Ans: A
! !! !"
! ! ! ! 2005
! !" !" !"
V = 30 ENTRANCE EXAMINATION TO SCHOOL
SCHOOL OF NURSING d. N42,000.0O
UNIVERSITY OF BENIN e. N50,000.00
TEACHING HOSPITAL, USELU, 4. Find the probability that a number
BENIN CITY selected at random from 41 to 56 is a multiple of 9
GENERAL NURSING PAST QUESTIONS AND
ANSWERS
INSTRUCTIONS TO ALL CANDIDATES
1. Your examination number (Not names) MUST be
written on your answer sheet
2. Indicate the option of your question paper answer
sheet
3. Submit both your answer sheet and question paper at
the end of the examination
4. to not write on the timing marked. Black shaded Bar
lines
5. Verify what has been shaded especially examination
numbers
6. Shade name properly. Erase thoroughly -
7. While writing leave space
1. Find correct to three (3) significant
figure, the value of (41830)
a. 205
b. 647
c. 6470
d. 64.7
2. The ratio of the price of a loaf to the price of sugar in
1975 was r: in 1980 the price of a loaf went up by 25%
and that of a packet sugar by 10% their new ratio is now
a. 10r:50t
b. 44r.40t
c. 50r44t
d. 44r:55t

3. Three brothers in a business deal share the profit at the


end of a contract. The first received 1/3 of the profit and
the second 2/3 of the remainder. If third received the
remaining N12.000.O0, how much profit did they share?
a. N60.000.00
b. N4,000.00
c. N8,000.00
a. 9. Which of the following solids has a
1/8 network structure?
b.
2/15 a. Diamond
c. b. Iodine
3/16
d. c. Sulphur
7/8 d. Graphite
e.
9/10 e. Iron
5. Solve the following equation and 10. Which of the following is likely to
find the value of x & y 2x+5y 0; 3x- have the highest degree of hardness?
219
a. 6&-
9 b.
4&0 c.
5&-2
d.
10&-4
e. 8&-1
6. If ABC is a right-angled triangle
with length of the side AB 6cm and
AC 9cm what is the length of side BC
a. 36
b. 81
c. 45
d. 15
e. 50

7. The quantity (x + y) is a factor of


a. x2 +y2
b. x3 + y3
c. 2x2 +3y2+y2—x+1
d. 2x3 + 2x2y - xy + 3x - y2
e. x5 + y5
8. The HCF of a2bx + ab2x and a2b-b3 is
a. b
b. a+b
c. a(a+b)
d. abx (a2 -b2)
e. (a-b)

CHEMISTRY
a. Carbonated water c. Combustion

b. Distilled water
c. Acidified water
d. Rain water
e. Lime water
11. Glucose can be obtained from starch
by
a. Hydrogenation
b. Dissolution
c. Hydrolysis
d. Dialysis
e. Dehydration

12. The following salts dissolves in cold


water except.
a. CaCI2
b. Pb504
c. (NH4)2 S04
d. Na2CO3
e. NaS03
13. In which of the following processes are large
molecules broken down into smaller molecules
a. Vulcanization of rubber
b. Hydrogenation of palm oil
c. Hydrolysis of starch
d. Polymerization
e. Esterification
14. Copper (II) tetraoxosulphate (IV) is often added to
swimming pools because it
a. Prevent the growth of algae
b. Coagulates suspended particles
c. Neutralize dissolved gasses in water
d. Reacts with any excess chlorine present
e. Increase the amount of dissolved oxygen
15. Methanol is obtained from wood by
a. Esterification
b. Bacterial decomposition
d. Fractional distillation 21. Which of the following PH values is

e. Destructive distillation likely to be that as a slightly alkaline


solution?
16. The scum formed when soap is a. 2
mixed with hard water could be
a. Calcium trioxocarbonate (IV)
b. Propane- 1,2,3- trio
c. Calcium
hydrogen
trioxocarbonate
(IV)
d. Magnesium tetraoxosulphate (IV)
e. Calcium stearate
17. What is the quantity of electricity produced
when a current of 0.5A is passed for 5hours 45
minutes (IF 9650CC)
18. In the reaction represented by the following
equation 2H25 + 502 — 2H20 + 35, 502 is acting
as:
a. A reducing agent
b. An oxidizing
c. A dehydrating agent
d. A bleaching agent
e. A precipitating agent
19. What is the number of oxygen atoms in 32g
of the gas? (0 16, L z6.0 x 1023 mo11)
a. 32 x 1023
b. 6.0 x 1023
c. 1.2 x 1024
d. 1.6 x 1024
e. 2.0 x 1024
20. The formular (CH3)3 CHO is
that of a a.Polyhydric alkanol
b.Secondary alkanol
c.Tertiary alkanol
d.Primary alkanol
e.Trihydric alkanol
b. 5 27…………. for the team for six years
c. 7 he decided to join another
d. 8 a. When he played
e. 13
22. Which of the following accounts for the difference in
the mode of conduction of electricity by metals and
aqueous salt solutions?
a. Electrons are present in metal but not in salt solution
b. Metals are conductors while salts are electrolytes
c. Electricity is carried by mobile electrons in metals but
by ions in aqueous salt solution
d. Salts ionize in aqueous solutions while metals do not
e. Metals are reducing agents while salts are not
23. When starch undergoes complete enzyme-catalyzed
hydrolysis the resultant products is
a. Glucose
b. Maltose
c. Sucrose
d. Fructose
e. Kerosene
24. Which of thefollowing is not a direct
petroleum product
a. Methane
b. Ethanol
c. Petrol
d. Vaseline
e. Kerosene

25. Hydrogen is used for the following


except
a. Manufacturing of Ammonia
b. Synthesis of hydrochloric acid
c. Extinguishing fire
d. Conversion of coal to petrol
e. Manufacture of margarine
26. Which of the following pollutants is
biodegradable?
a. Sewage
b. Plastic
c. Metal scraps
d. Lead compound
e. Hydrogen sulphide
Choose the expression which best complete each sentence
in question 27-37
b. Played but I did not know exactly
c. Having played a. Where are you going
d. In playing b. Where you were going
28. The bridge connecting the two
cities was by the enemy
a. Blown up
b. Blown down
c. Blown off
d. Blown over
e. Blown
29. My little boy is
suffering jaundice
a. From
b. Off
c. For
d. With
e. By

30. They went to the market and


bought a suit case and bag
a. A big leather brown
b. A leather brown big
c. A big brown leather
d. A brown big leather
e. A leather big brown
31. By the end of this year
in this town for eleven
years
a. I‘m living
b. I‘d be living
c. I‘ll live
d. I‘ II have lived
32. If he had left home earlier he
--late
a. Would not be
b. Would have not been
c. Would not been
d. Would have been not
e. Would not have been
33. I saw you walking in that direction
c. Where were you going to c. 70Cm

d. To where were you going d. 250Cm

e. Where you are going


34. The hill behind the town was so steep
a. That no man could find a foot holds on
b. That any man could find a foot hold on it
c. For a man to find a foot hold on
d. That no man could find a hold. on it
e. For any man to find foot hold
on

35. Walking down the street


a. The boy was knocked down by
b. A car knocked the boy down c. knocked down the car
d. A car collided with the boy
36. The project which seemed very near
realization, because of lack of funds
a. Fell over
b. Fell through
c. Fell off
d. Fell in
e. Fell down
37. The chairman of the company was
very sympathetic his staff
a car
a. To
b. At
c. For
d. From
e. On
PHYSICS
38. A machine has an efficiency of 60%. If the machine
is required to overcome a load of 30N with a force of
20N, calculate its mechanical advantage
a. 0.7
b. 0.9
C. 1.5
d. 2.5
e. 10.0
39. A bus traveling at 15ms-1 accelerates uniformly at
4mc-2, what is the distance covered in 10s?
a. 35Cm
b. 15Cm
e. 600Cm
44. A converging lens of focal length 5cm
40. Which of the following media allow(s) the is used as a magnifying glass by a man
whose near-points is 2. Calculate the
transmission of sound wave through them? I Air magnification given by the lens.
II. a. 8
b. 7
Liquid III. Solid
a. I only
b. II only
c. III only
d. I &II only
e. I,II & I!I
41. Which of the following wave characteristic cart be
used to distinguish a transverse wave from
longitudinal wave?
a. Reflection
b. Refraction
c. Diffraction
d. Polarization
e. Interference
42. Which of the following is
not a product of nuclear fission?
a. Alpha particle
b. Beta particle
c. Gamma-ray
d. X-ray
e. Neutron
43. Calculate the power delivered by a
3- phase line if its voltage and current
are 132kv and
respectively. a. 2.38 X
107w
b. 7.92 X
106W c. 3.96
X 106W d.
2.64 X 104W
e. 7.92 x
4
10 W
c. 6 a. Long neutral spine
d. 5 b. Odontoid process
e. 4
45. Which of the following are both
mechanical and transverse?
a. Infra-red rays
b. Gamma rays
c. Sound waves
d. Water
e. Micro waves
46. A ray of light strikes a plane mirror
at a glancing angle of 50‖. Calculate
the angle between the incident and
reflected rays
a. 20° -
b. 40°
c. 50°
d. 80°
e. 1000
47. Which of the following is not a
component of a d.c electric motor‘
a. Commutator
b. Field magnets
c. Slip rings
d. Armature brushes
e. Carbon burshes

MATHEMATICS
48. Find the simple interest rate percent
per annum at which N1000.00
accumulates to N240 in 3 years
a. 6%
b. 8%
c. 10%
d. 12%
e. 14%
49. The area of a square is 144cm2, find
length of its diagonal
a. 11 3
b. 12cm
c. 12 2
d. 13cm
e. 13cm
50. Which of the following levels of organization in
living things in the correct sequences, starting from the
most complex to the simplest?
a. Tissue -Cell - Organ - System
b. System - Organ - Tissue - Cell
c. Cell - Tissue - Organ - System
d. Cell - Tissue - Organ - System
e. Organ - System - Tissue - Cell
51. The thoracic vertebra differs from the other vertebrae
by the possession
C. Vertebraterial canal c. Adeowale‘s own
d. Large Centrum
e. Transverse process

52. The control of water and salt requirement of the


body in order to stabilise internal environment is
known as
a. Osmosis
b. Excretion
c. Plasmolysis
d. Ultra filtration
e. Homeostasis
53. The part of the ear that equalizes air pressure
on either side of the ear drum is the
a. Auditory meatus
b. Melleus
c. Eustachian tube
d. Oval window
e. Stapes
54. Which of the following
determined kline felter syndrome
a. X
b. Y
c. XXX
d. XXXY
e. YY

ENGLISH LANGUAGE
55. Adesuwa is sure to pass in November. She hard
since the beginning of this term
a. Had been working b. was working
c. Is working
d. Would work
e. Has been working

56. Do you know if that dog is


He has lost a brown and black one
a. Adeowale‘s
b. Adeowales
d. The Adeowales d. Dupe me

e. The Adeowale‘s
57. The students in my junior class seem to have
performed they did last year
a. Better than
b. More better than
c. Most better as
d. As better than
e. Least better than
58. After the Lagos bomb blast, our association received
many letters from those adversely affected and we
resolved to
a. Assist them
b. Invite them c. ftespond them
d. Pity them
e. Query them
59. The story has to be taken with a pinch of salt. The
means that
a. You need some salt to listen to the story
b. There is no salt the story
c. The story is questionable
d. The story is true
e. You have too much salt in the story

60. It is usually hard to change the course or action when


one crosses the rubicon. The underlined expression as
used in this sentence means to
a. Pass though a place called rubicon
b. Cross a river called rubicon
c. Cross a bridge called rubicon
d. Pass a special test
e. Be irrevocably committed.
61. The salesman tried to pull the wool over my eyes.
This implies that the salesman tied to
a. Force me to buy his goods
b. Offer me cotton wool
c. Make me buy his wool
e. Cover my eyes with wool right of the left of men and if the control breaks down,
man doesn‘t hesitate to use force or to destroy life. The
62. Oboh counted his chickens before they were hatched. end, the saws justifies the means and men are only of
This means that Oboh
a. Regarded each egg as a chicken
b. Hatched the eggs prematurely
c. Assumed that her expectations had already
been realized
d. Protected her eggs from breaking
e. Insured the egg.
63. Adamu has resigned his job with the Textile mill.
He doesn‘t seem two
worry about getting another job his plans are still
quite in the air. This means is plans are
a. Airmail
b. Air tight
c. Uncertain
d. Certain
e. Airborne
Read the passage carefully and answer question
72- 76 that follow it by shading the right option
At the time of the troubles in Ireland. A priest said:
―Yes and the beast; A diplomat replied; Yes and the
beast is the half I like the best. The priest meant that
few beasts behave as badly as man when men make up
his mind to behave badly. If you look at the folly and
cruelty of today‘s world, it is hard to disagree with the
diplomat or priest. But human nature can be changed.
Anybody, if that is what they most want to do, can
changed the most difficult person they known. The art
of changing people has been lost in the modern world.
That is why the modern world has lost its way.
There are two ways of looking at human nature. One is
to make the best of its as it is and assume that it is the
raw material of life which cannot be altered. That is
what most people in the free world do today in these
circumstances; if you expect the worst you are seldom
disappointed. Faith today has become irrelevant to the
everyday needs of so many people in position of
responsibility because they do not expect faith to
change men.
Another way of dealing with human nature
is to exploit it. All materialists, whether of
the
value in so far as they are a mean; towards the 69. The last editions of the National
achievement of his ambition. If they cannot be bribed or Sports Festival took place in
forced to play their part, then they must be liquidated.
64. The Priest meant that
a. Man is beast that human
b. Man could be worse than a beast if he want to
c. He liked beast more than man
d. He liked both man and beast
e. Really, a man cannot be beast.
65. The modern world has lost its way because
a. It no longer believe in God
b. Ambition has made people greedy
c. People are no longer interested in one another
d. The technique of influencing people is no more
e. People are just too selfish
66. What most people do in the free world today is to
a. Live other people alone
b. Assumed that human nature is
c. Accept human nature as it since it is immutable
d. Attempt to change the world
e. Leave the world as it is

67. Faith today is by and large irrelevant because


a. Nobody thinks human nature
b. Most people do God
c. Being in a position responsibility nullifies
importance of faith
d. Responsible people need not bother about faith
e. The faithful are very few in the world anyway.
68. Materialist can be found
a. In capitalist countries
b. In communist countries
c. In less developed countries
d. Among people who do not have neighbor
e. All over the world.
a. GENERAL c. Chief Adolphus Wabara

b. Abeokuta
c. Lagos
d. Port-Harcourt
e. Abuja
70. The name of the Head of service
of the Federation is
a. Aihaji ban. Suleiman
b. Alhaji ban. Usman
c. Alhaji Mohammadu Ahmed
d. Chief Rufus kaete
e. Chief Adolphus Wabara

71. The name of the Head of wealth of nations


who visited last year is
a. Tony Blair
b. Queen Elizabeth I.
c. Queen Elizabeth II
d. George Bush
e. Margaret Thatcher
72. Which of the following parastatals was judged
the performance last year?
a. NUC
b. NAFDAC
c. UCH
d. SAVAN
e. NPA
73. The names of the super eagles coach
a. Mr. Obakpolo
b. Mr. Joe Asien
c. Christian Chukwu
d. Mr Christian Chukwuma
e. Mr. Joe Udubor
74. The name of the senate president is
a. Mr. Kojo Waner
b. Alhaji Bello Mason
d. Obong Ufot Ekaete b. Deoxygenated blood from the lungs to the right atrium

e. Alhaji El-Rufai c. Oxygenated blood from the lungs to the right atrium

75. The name of the minister of health is


a. Prof. Eyitayo Lambo
b. Prof. ABC Nwosu
c. Prof. EE Okpere
d. Prof. BC. Ayodele
e. Prof. Idris Mohammed
76. The GENERAL TEACHING teaching
hospital was officially opened on
a. 12th May, 1973
b. 12th May, 1972
c. 12th May, 1970
d. 12th May, 1982
BIOLOGY/HEALTH
77. EPI is useful in the prevention of
a. Congenital abnormalities
b. House trauma
c. Early childhood deadly
d. Disease
e. Six childhood killer disease
78. Which of the method for
INFECTION?
a. Safe sex
b. Use of condoms during sexual intercourse
b. Eating from the same plate with an HIV patient
d. Kissing
d. Blood transfusion
79. Which of the following is not a function of the
skeleton?
a. Providing support for
b. Controlling growth animals
c. Maintenance of the shape of the body
d. Protection of the internal delicate organs
80. The pulmonary veins carries
a. Oxygenated blood from the right ventricles to lungs
d. Oxygenated blood from the right ventricle to the lung a. Blood
81. Which part of the brain is concerned with b. Serum
regulation of heartbeat, breathing rate and body c. Protoplasm
temperature? d. Plasma
a. Cerebellum e. Lymph

b. Medulla 87. Which of the following organs is

c. Cerebrum associated with decimation of

d. Olafactory Lobe
d. Cortex
82. Which of the following is
not a component of mammalian
skin?
a. Epidermal cells
b. Nerve-endings
c. The subcutaneous fatty layer
d. Sebaceous glands
e. Hepatocyte
83. Which of the following is the correct route
through which the ovum passes from the ovary in
the vagina?
a. - Ovary - Cervix - Fallopian tube
- Fallopian - Funnel Vagina
b. Ovary - Fallopian - Tube Uterus - Cervix Vagina
c. Ovary - Fallopian- Tube - Certvix- Uterus Vagina
d. Ovary - Cervix -
Uterus Fallopian -
Tube - Vagina
e. Ovary - Fallopian -
Tube Fallopian - Funnel
- Cervix Vagina
84. Transmission of malaria
parasite is through the bite of the
a. Male Anopheles Mosquito
b. Female Anopheles Mosquito
c. Infected male Anopheles mosquito
d. Infected Aedes Mosequito
e. Infected female Anopheles mosquito
85. How many chromosomes are
found in the human ovum?
a. 46
b. 23
c. 33
d. 13
e. 43
86. Which of the following is the medium of
transportation of nutrients within unicellular
organisms?
proteins? Profit of second = 2 3 (x - 3)
a. Lungs = 2 3 - 2 9 = 4 9profit of third = x [ 3 + 4 9]
b. Stomach =x-29

C. Kidneys ∴ if 2 9 = 12,000
2x = 9 x 12,000 = 108,000
!"#,!!!
d. Liver ∴ x= = N 54,000
!
e. Heart
4. Ans: A. multiple of 9 between 41 and 56 are 45 and
88. Ultra filtration in the kidney takes
54 (2 items)
Place in the Total numbers between 41 and 56 is 16.
a. Loop of Henley ! !
∴ pr(multiple of 9) = =
b. Renal vein !" !

c. Bowmen‘s or glomerular 5. Ans: C. Sec Q. 24 of year 2004


capsule
6. Ans: C
d. Pelvis
e. Pyramid
89. Conventional method of sewage disposal involves all
the following except defecating
a. Into pit latrines
b. In open fields
From Pythagoras‘ theorem,
c. Into Cesspit
/AC/2 = /AB/2 + /BC/2
d. Into bucket latrine ∴ /BC/2 = /AC/2 - /AB/2
/BC/2 = 92 – 62 = 81 – 36 = 45
e. Into water borne system
/BC/ = 45
90. Which of the following is not an 7 Ans: B
example of a simple reflex action? 8 Ans: A
a2bx + ab2x = ab(ax + bx)
a. Knee jerk a2b – b3 = b(a2 – b2)
b. Blinking of the eye comparing both equation, HCF = b

c. Withdrawal of the hand from hot object 9 Ans: D


d. Reading a book Graphite has a layered structure Iodine has a
harring-bone shape
e. Contraction of the Iris
10 Ans: E
ANSWERS TO 2005 NURSING QUESTIONS Lime water contains plenty of calcium ions, which
1. None of the options is correct. 49 is written to three cause hardness of water.
(3) significant figure as 41800
2. Ans: C 11 Ans: C
10
new ratio = r + 25 100 : t + 100 Hydrolysi breaks down large molecules into
= r + 4 : t + 10 Profit of first = 3
= 5 4: 11 10
= 50r : 44t

3. None of the options is correct!


Let the total profit be N x
smaller ones. The reverse
process of condensation.

12 Ans: B
Pbso4 is an insoluble salt

13 Ans: C
See Q. 111 above
14 Ans: A 26 Ans: A
15 Ans: E Biodegradable pollutants can be broken down into
Method is i=otherwise called WOOD SPIRIT harmless substances by living organisms

16 Ans: E
Soap is sodium stearate (C17H36COONa)
2C17H36COONa + Ca2+
→( C17H36COO)2Ca
+2Na+ 27 Ans: C
Scum ―Having played‖ because in a participial clause
structure, the subordinate clause could be
17 Q = 1 x t. introduced with having + past participle verb
Q = 0.5 x (5hrs x 60mins)+45min before the main clause.
Q = 0.5 x 345mins
28 Ans: A
Q = 0.5 x 345mins
―Blown up‖ means to destroy something by an
Q = 0.5 x 345mins x 60 seconds.
explosion
Q = 10,350C ()0.107F)
29 Ans: A
―Suffering from‖ means to be badly affected by a
disease (jaundice) or pain, sadness, or a lack of
18 Ans: B something
SO2 is a reducing but can also behave as an 30 Ans: C
oxidizing agent in the presence of a stronger agent In the order of adjective; determines (a) comes
such as H2S. first, followed by size (big), colour (brown), then
material (leather) plus noun head (bag). ―a bag
19 Ans: C brown leather bag)
32g of O2 = 1 mole of O2 molecules 31 Ans: D
= 2moles of atoms ―I have lived‖ because when expressing the future,
but 1 mole = 6.02 x 1023 we use the present tense to express what will
∴ 2 mole of O atoms = 2 x 6.02 x 1023
32 Ans: E
1024 atoms
―Would not have been‖ this means he was late
because he did not leave home earlier
OH
20 33 Ans: B
―Where you were going‖ because clausal adverbs
C
do not the SVOCA structure to denote place,
CH3 CH3
direction or location
34 Ans: D
―That no man could find a hold on‖ in a clausal
TertiaCrHy3 alcohol (alkanol)
35 Ans: A
21 Ans: D applicable of this purpose
Alkalinity increases from after a PH of 7

22 Ans: C

23 Ans: A
See Q 11 above

24 Ans: B

25 Ans: C
Hydrogen burns in air and will not be suitable as
fire extinguisher. Carbon (IV) oxide is more
―The boy was knocked down by a car‖ because in a participial clause
structure, the subordinate could be
introduce with a gerund (walking
down the street) before
the main clause
36 Ans: B
―Fell through‖ means ―failed‖ or ―became a
failure‖

37 Ans: A
―Sympathetic‖ collocates with ―to‖ or ―towards
somebody‖ meaning to be kind to somebody who
is hurt or sad; showing that you understand and
care about the problems
38 Ans: C
!"#$ = !"!= 1.5
M.A =
!"#$% !"! System
39 Ans: A
U = 15m/s a = 4m/s2 t= 10s Organ
S = ut + 1 2 at2
S = 15(10) + 1 2 (4)(10)2
S = 150 + 200 = 350cm. Cell
40 Ans: D
Organelle
41 Ans: D
Longititudinal waves can be polarized whereas 51 Ans: A
transverse waves cannot Thoracic vertebrate are found in the chest region.
The odontoid process is part of the axis vertebra.
42 Ans: A 52 Ans:E
53 Ans: C
43 None of the above The Eustachian tubes connect our ears to our
Power = I x V throat
Current wasn‘t given 54 Ans: C
44 Ans: B Xxx female with klinefelter‘ syndrome
Xxy = male with klinefelter‘ syndrome
45 Ans: C Klinefelter‘ syndrome is also called TRISOMY –
x
55 Ans: E
46 Ans: D
―Has been working‖ because it is used to express
Sum of angles on a line = 180
what started in the past that is still on going
∴ angle between rays = 180 – (50=50)
= 180 -100 = 800 56 Ans: A
―adewales‖ possessive case noun do take
apostrophes when it denote singular ownership
47 Ans: C
with proper nouns.
48 Ans: B 57 Ans: A
!!!!! ―Better than‖ Used in comparison between two or
I= more person or things
!""
!""! !"" ! !"# 58. Ans: C
R= = = 8%
! ! ! !"" ―Assist them‖ means to help some persons to do
something
49 Ans: C 59. Ans: C
Area = L x B (B = L for a square) Sometimes, we write:
144 = L x B = L3 Organism
L = 144 = 12
A

12

B C
From Pythagoras theorem
/AC/= 12 ! + 12 !
/AC = 144 + 144 = 288
/AC/ =

50 Ans: B
―To take with a grain of salt‖ or
? A grain of salt means ―the
story is questionable‖
60. Ans:E
―To cross the rubican‖ means
―be irrevocably committed or not
turning back‖

61. Ans:D
―To pull the wool over my eyes‖
means ―to dupe me‖

62. Ans: C
―To count ones chicken before
they are hatched‖ means
―assumed that her expectation
had already been realized‖

63. Ans: C
―To be in the air‖ means ―to be
uncertain‖
64. Ans: B lungs to left atrium
“Man could be worse than a beast if he want to‖
because man‘s action can no longer be tamed like
that of the beast

65. Ans: C
―Ambition has made people greedy‖

66. Ans: C
―Accept human nature as it is immutable‖ this is
because one is to make the best of it. As it is and
assume that it is the raw material of life which
cannot be altered

67. Ans: A
Nobody thinks it can alter human nature‖ because
if you expect the worst or the best you are seldom
disappointed

68. Ans: E
―All over the world‖ whether of ―the right or left‖
meaning ―everywhere‖

69. Ans: E
70. Ans: B
He was in office between 2003 and 2007
71. Ans: A
He was the Britain prime minister

72. Ans: B
Prof. Dora Akunyili was the boss of the agency as
of the time

73. Ans: C
He was formerly a player for the Nigerian national
team

74. Ans: C

75. Ans: A
Prof. E. Eugene Okpere (professor of obs. ∝
Gynea) was the chief Medical Director of UBTH

76. Ans: E
It was established in 1973

77. Ans: C

78. C, D and E are all correct

79. Ans: B

80. Pulmonary veins carry oxygenated blood from the


81. Ans: B
The medulla oblongata controls involuntary
actions and body temperature (via
vasoconstriction and vascodilataion).
82. Ans: E
Hepatocytes are found in the liver

83. Ans: E

84. Ans: E

85. Ans: B
86. Ans: C

87. Ans: B
The enzymes bringing about the change in the
stomach is called PEPSIN.

88. Ans: C
Bowman‘s capsule + glamerulus = malpighian
body

89. Ans: B
90. Ans: D
Reading a book 100% voluntary
2004 day
ENTRANCE EXAMINATION TO SCHOOL c. People from all works of live attended the
OF NURSING UNIVERSITY OF BENIN ceremony
TEACHING HOSPITAL, USELU, BENIN
CITY GENERAL NURSING PAST
QUESTIONS AND ANSWERS
ALTERNATIVE A TO ALL CANDIDATES:
1. Your examination number (Not Names) must be
written on every of the question paper which is also
your. Answer booklet
2. To the left of the Options Mark ―T‖ for Correct and
―F‖ for correct
3. Wrong answers will attract negative marks

Read the following passage carefully and answer the


questions follows
An educated man, as an educationists , is one with an
all-round training intellectual aspect, which the church
attends to the moral aspect. It is through co-curricular
activities that physical and social aspects will be
adequately covered co-curricular activities expose a
child‘s innate, talents will be adequate games. Farming
dressmaking and so on. They enable him join societies,
clubs where he develops socially. His talents can then be
tapped for the benefits of his country and the larger
society.
There is need for co-curricular activities to be made part
of the school curriculum at all levels. besides developing
a child‘s talent, co-curricular activities can be used by
institutions of fund generating at this austere period they
provide relaxation periods for students, for all work and
no play makes jack a dull boy.
I will advocate that all hands be on deck to see to their
successful implementation. Government, education
ministers, school administrators, parents should play
vital roles in their implementation and sustenance.

1. Based on the passage it suggests


a. The need for co-curricular
b. The central point being stressed is that
government should introduce sustain cocurricular
c. Austere‘ in the passage means
d. The tone of the passage is analytical
2. Concerning collective nouns
a. A troop of army
b. A host of scholars
c. A pride of lions
d. A troop of monkeys
3. The following sentences and spelling are
correct
a. If you don‘t want to lose your watch don‘t go
there
b. He was to give a talk on Monday, but
unfortunately, he had an accident on the faithful
d. It is unnecessary and embarrassing to come a. An acid changes red lithmus paper blue
in at that point b. An acid is a proton acceptor
4. Concerning appropriate use of words,
is the underlined word appropriately used?
a. I have told you many times that you
are a wicked woman
b. .Nigerian men like to wear
colourful dresses c.He tried to gain
admissi
on to the university
d. Newton gravity

ALTERNATIVE A:
5. Occupation and names
a. A woman who write books is an authoress
b. A person who shoots with bows and arrows is
an archer
c. A person who sells fish is a fish monger
d. A woman who cooks in a restaurant is a chef
BIOLOGY
6. human skin is
a. For protection
b. The largest organ in the body
c. Has 4 layers
d. Involved in
vitamin production
7. In the human skeleton
a. The largest joint is the knee
b. The olecranion process is part of the radius
c. The elbow joint is a ball and socket joint
d. The gliding joint possess synovial membrane
8. Concerning blood vessels
a. The pulmonary artery carries deoxygenated blood
b. The pulmonary vein carries deoxygenated blood
c. The portal vein drains from the heart
d. The intima is the middle layer of the wall of the artery
9. In the human blood grouping system
a. A person with blood group 0 is a universal recipient
b. A person with blood group AB is a universal recipient
c. A rhesus negative mother carrying a rhesus
positive foetus has no risk
d. Blood group AB can be given to a person with
blood group A
10. In the human gastro-intestinal system
a. Pepsin is necessary for carbohydrate digestion
b. Gastrin is necessary f or protein digestion
c. Amylase is secreted for the emulsification of fat
d. Bile salts are products of digestion
11. Concerning acids and bases
c. Bases are compounds that liberate hydroxyl ions in 20. Concerning states in Nigeria and their appellations
solution a. Kwara state: the confluence state
12. An element has a mass number of 23
and an atomic number of 12
a. It has 10 electrons
b. It has 12 neutrons
c. It has 6 electrons in the outer
d. It has 23 protons

13. The following concern mixture and compound


a. Sublimation as a method of separating compounds
b. Energy is not required in the processing of crude oil
c. Fractional distillation is used in the processing of
crude oil
d. Blast furnace is used in. the purification of Iron
ALTERNATIVE
14. Concerning gases
a. Oxygen rekindles a glowing splint
b. Hydrogen is the heaviest gas
c. Sulphur-2-oxide (502) has the smell of Rotten Egg
d. Carbon monoxide (CO) does not support combustion

PHYSICS
15. Consider a ball falling from point F to the ground
a. The potential energy at point Fiszero
b. The kinetic energy at point F is at if S maximum
c. The potential energy at ground level is zero
d. The kinetic energy just before it hits the ground mgh
16 The following are true
a. Gravitational force is a form of field force acting on
other bodies across a distance without making contact.
b. Gravity is the force with which the earth attracts
objects towards it‘s centre
c. Light energy travels in a material medium
d. Short sightedness is corrected using convex lens
17. Concerning machines
a. First class levers have the fulcrum between the effort
and load
b. Second class levers have the effort between load and
fulcrum
c. Efficiency=mechanical advantage/velocity
d. Kinetic energy is energy possessed by a body by
virtue of it‘s position
18. Concerning electrical circuits
a. The current is measured with a voltmeter
b. The potential difference is measured with an Ammeter
c. The ticker a wire, the faster it conducts electricity
d. In a parallel circuit, the current divides and pass
through each conductor separately
19. The federal republic of Nigeria
a. Was a former American colony
b. Became independence on 1 October 1961
c. Became a republic 1St Oct.
d. Has an area of 923, 768.68km
b. belta state: the heart beat of the nation B true “the central point being stressed is that
c. Kogi state: state of harmony government should introduce sustain co-
d. Edo state the heart beat of the nation curricular” in the last paragraph, it reads
government, education minister, school
administrators, parents should play roles in their
implementation and sustenance, this is also based
on the emphases on the first paragraph

ALTERNATIVE A
21. The GENERAL TEACHING
b. The chairman of the board of management in has
Hon Justice Nwazoto
c. The Chief medical director is Professor Oronsaye
d. The tirector of
Administration is Mrs.
Idubor
22. Concerning the Nigerian 4th republic
a. The NP presidential candidate was the
Nwachukwu b The House of Representative is led
by Aldophus Wabara
c The senate is led by Bello Massari
d. Gani Fawchinmi was presidential
candidate democratic alternative

MATHEMATICS
For question 23 to 26 circles the correct option
23. If 14 is added to 2/3 of a number the result
11/4 times the original number what is the
number?
a. 14 b. 12 c 24 d. 15
24. Solve the following equation and find value of x
and y
2x + 5y
0 3x -
2y: 19
a. 6and4 b. 4and0 c. 5and-2 d. 10 and -4
25. Average speed of 50.5km/hr of
car A and 30.5km/hr for car B, what
the distance between the two cars
after
31/2 hours?
a. 80.70km b 160.70km c 106.0km d. 310.00KM

ANSWER TO 2004
NURSING PAST
QUESTIONS
1. True A. “The need for co-curricular” the first
line in the second paragraph needs: there is need
for co-curricular activities to be made part of the
school curriculum at all levels. The benefits are
he/she will develop socially, it is used to
provide relaxation for students.
C. true “austere” in the passage means ―hard‖ in 6. A. true B. True C. False D. False
paragraph two line 2 it reads besides developing a
child‘s talent, co-curricular activities can be used
by institution of fund generating at this austere
period
D. true “the tone of the passage is analytical”
which means explanatory as well as expository
through step method and thereby stating its
benefits concerning collective nouns
2. A. True “a troop of army” which means soldiers,
especially in large groups
B. true “a hast of scholars” denote a large
number of people or things
C. true. “a pride of lions” denote a group of
monkeys. The following sentences and spellings
are correct
3. A false. If you don‘t want to lose (loss) your
watch don‘t go there ―loss‖ means the state of no
longer having something or as much of something
while ―lose‖ means to be unable to find something
or somebody
B. false. He want to give a talk on Monday but
had an accident on the faithful (fateful)
Day. NB ”faithful” means staying with or
supporting a particular group, person or
organization or belief, while ―fateful” means
having on important often very bad, effect on
future event
4. Concerning appropriate use of words, is the
underlined word appropriately used
A. True. I have told Mary times that you are a
wicked woman
B. False. Nigerian men like to wear colourful
dresses (shits). NB ―dress‖ are clothes ie. S
piece of women‘s clothing while ―shirts‖ are
for men
C. True. He tried to gain admission t the
university
D. False. Newton invested the law of gravity.
NB. Sir Isaac Newton discovered the law of
gravity and not invented
Alternative A
5. A. True. A woman who write books is an
authoress
NB. The word ―authoress‖ is an old English the
modern usage for both male and female is called
―author‖
B. True. A person who shoots with a bow and
arrows is an archer
C. true. A person who sells fish is a fish manger
D. true. A woman who cooks in a restaurant is a
chef
NB: ―Chef‖ means a professional cook, especially
the most senor cook in a restaurant, hotel etc.
There are 3 layers in the skin Viz. cornified measures potential difference.
Layer (stratum corneum) 19. A. False B. False C. False D. True
Granular Layer (stratum Granulosum)
Malpighian/germinative layer
(stratum
germinosum)
The skin is involved in Vitamin D production.
7. A. False B. False C. False D. True
The elbow joint and knee joint are hinge joints.
The shoulder and hip joints are ball a d socket
joints
8. A. True B. False C. False D. False
All arteries carry oxygenated blood except the
pulmonary arteries. All veins carry
deoxygenated blood except the pulmonary vein.
All arteries (no exception) carry blood away
from the heart. All veins (no exception) carry
blood toward the heart

9. A. False B. True C. False D. False


O is the universal donor while AB is the
universal recipient
10. A. False B. True C. False D. False
Pepsin and trypsin digest proteins ptyalin and
amylopsin are amylases (i.e they digest starch)
Rennin curdles milk
Gastrin stimulates gastric juice secretion
11. A. False B. False C. true D. False
Acids are proton donors, liberate hydrogen ions
in water and turn blue litmus paper red. Bases
are proton acceptors, liberate hydroxyl ions in
water and turns red litmus paper blue.
12. A. False B. False C. False D. False
Proton number = 12
Neutron number = 23 – 12 = 11
Electron number = 12
Electrons in outer shell = 2 (from 2,8,2)
13. A. False B. False C. True D. True
Sublimation is a separation technique for
mixture. Components of mixture can be
separated but those of compounds cannot
14. A. True B. False C. False D. False
Hydrogen is the highest known gas
Hydrogen sulphide (H2S) has a rotten egg smell
15. A. True B. False C. False D. False
Kinetic energy is maximum just before in hits the ground
16. A. True B. False C. False D. False
Convex lenses correct long sightedness
(hypermetropia) short sightedness (myopia) is
corrected using concave lenses
17. A. False B. False C. True D. False
!"#!!"#$!% !"#!$%!&' !""
Efficiency = x
!"#$%&'( !
18. A. False B. False C. False D. True
Ammeter measures current while voltmeter
Nigeria was a Britain colony that became 1. The Rwanda massacre claimed 800,000 lives. Which
independent in 1960 and became a republic in tribes most severely affected
1963 a. The Ibos
20. A. False B. False C. False D. True b. The Tutsi
The heartbeat of the nation-Edo state confluence c. Moderate Hutus
State-Kogi state, the big heart delta state d. The Hausas
e. The Hard-Line Hutus
21. A. - B. True C. False D. False 2. The following leaders of the federal legislative house
Chief medical director is a term that applies to were impeached in the 4th republic
UBTH and not to Uniben a. Ayim and Buhari
22. A. True B. False C. True D. False b. Qkadigbo and Na-Abba
Gani fawehinmi contested for the presidency c. Ayim and Na-A bba
under the National Conscience Party (NCP) d. Ewerem and Okadigbo
23. A. False B. False C. False D. False e. Ewerem and Buhari
3. The following states have had their speakers
Let the number be x
impeached at various times during the 4th republic.
Then 14 + 2 3 = 11 4 !
a. Delta
!C! olle!ct like terms
x - x = 14 b. Oyo
! ! c. Edo
!!!
-
!!
= 14 ii. Mark ―T‖ for the correct answers and ―F‖ for the answers
!"! that are not correct
= 14 iii. Wrong answers will attract negative marks
!"
25x = 14 x 12 = 168
!"#
X=
!"
None of the option is correct.
24. A. False B. False C. True D. False
2x + 5y = 0 x 3 6x + 15y=0 →(1)
3x + 2y = 19 x 2 6x + 4y = 38 →(2)
Subtract 2 from (1)
6x + 15 = 0
-6x – 4y = 38
=19y = -38
Y = −38 19 = -2
Substitute -2 for y in (2) above,
6x—4y=38
6x - 4(-2) = 38
6x+8=38
6x=38—830
= 30/6 = 5
:.x=5and y=2
25. A. False B. False C. False b. True

JUNE 2003
ENTRANCE EXAMINATION TO SCHOOL
OF NURSING UNIVERSITY OF BENIN
TEACHING HOSPITAL, USELU, BENIN
CITY
-
Option “C’,
INSTRUCTION TO ALL CANDIDATES
i. Your examination number (Not Names) must be
written on every page of your question booklet.
d. Osun
e. Kaduna
4. The following are either
Palestinian or Israeli leaders
a. Hosrii Mubarak
b. Sadam Hussein
c. Yasar Arafat
d. Ariel Sharon
e. 5himon Peres
5. The Nigerian senate has
………senators a. 190 b. 360 c. 119
d. 306 e. 160
6. The following African
countries qualified for the
finals of the 2002 world cup
a. Ghana b. Egypt c. South Africa
d. urkina Faso e. Senegal

7. The following politicians


were jailed by the military
a. Chief A. Annenih
b. Chief Jim Nwobodo
c. Chief Olusegun Obasanjo
d. Chief Gani Fawchinmin
e. Dr. 0. Ogbmudia
8. The following
constitutional cases were
recently decided by the
Supreme Court.
a. Resource control
b. Tenure of
elected local
government
officials
c. Tenure of elected Governors
d. The murder of Chief Bola Ige
e. Registration of new parties
9. Sharia is being
practiced in the following
states
a. Zamfara
b. Kano
c. Niger planned between:
d. Sokoto a. Rhesus positive man and Rhesus negative woman
e. Kebbi
10. The following are concerned with the regulation of
nursing practice in Nigeria
a. Medical and Dental council of Nigeria
b. Pharmaceutical council of Nigeria
c. The nursing and midwifery council of Nigeria
d. The West African College of Nursing and midwifery
e. The National Association of Nigeria nursing and
midwives
11. Water diffuses through a
hydrophobic membrane with ease because
a. The water molecule
dissolve in hydrogen
b. The water molecules are small
and uncharged
c. The water molecule are liquid
and small
d. All of theabove
e. None of the above
12. Hydrogen peroxide is a destructive by-product of
cell metabolism and so the cell isolates it and breaks it
down in the
A .Mitochondria b. VacuoIes
C Iysosomes d. a & c e. All of the above
13. Acquired immune deficiency syndrome can be
transmitted by sexual the Following a. Sexual
intercourse
b. Through blood transfusion
c. From a pregnant mother to the unborn child
d. Through the sharing of needles
e. Through eating from the same
14. The following ducts drain directly into
the lumen of the small intestine
a. The pituitary gland duct
b. The bile duct
c. The splenic duct
d. The pancreatic duct
e. The duct of the salivary glands
15. A person that is obese should avoid
eating food containing a lot of
a. Yam
b. Carrots
c. Bread
d. Beef
e. Orange
16. Some of the adaptation of life on trees by monkey is:
a. Its possession of a long tail
b. Its possession of long canine teeth
c. Its ability to frighten away birds
d. Its possession of lông digits
e. Its herbivorous nature
17. Genetic counseling is important in a marriage
b. A Hb genotype AS marrying an AC c. The number of protons in the nucleus is the atomic
c. Rhesus negative man marrying a Rhesus number
negative woman
d. A Hb Genotype AS man marrying an AS
woman All of the above
18. The following organs are located in following
body compartments
a. Kidneys and heart in the chest
b. The heart and lungs in the abdomen
c. The kidneys and ovaries in the abdomen
d. The pituitary and thyroid glands in the brain stem
e. The pancreas and lungs in the chest
19. The animal cell
a. Has a characteristic cellulose cell wall
b. The chromosomes are a major component of
the nucleus
c. Chloroplasts may be present in the cytoplasm
d. Mitochondria are in the vacuoles
e. RNA are present in both the cytoplasm and
the nucleus
20. The hormone which tones up the muscles of a
person in times of fright and danger is from
a. The thyroid gland b. Pancreas
c. The adrenal gland d. The liver
e. The spleen English
In each of the following sentences, there is one word
underlined. From the list of words lettered A to E, mark
―T‖ for any word that has the same meaning and ―F‖ for
a word that is not the some meaning.
21. bespite babatunde‘s insistence that his pistol went
off accidentally, the police still believed that he acted
deliberately
a. Purposefully b. Carelessly c. Unknowingly
d. Accidentally e. Willfully
22. The reply of the accused to the question was
careless rather than cautions.
a. Rude b. Scrupulous c. Carefree
d. Hasty e. Thoughtful
23. The guidance counselor
discovered that his students were
knowledgeable whereas he had
thought them to be dull
a. Hard working
b. Intelligent
c. Already quarreling with
each other
d. Rearing cats and dogs
e. Avoiding each other Chemistry
24. Regarding the arrangement
of electrons in the atom
a. The maximum possible number of electrons in a
shell numbered n is 2n2
b. Maximum no of electrons in the outermost shell
of any atom is 28
of the element
d. The energy in each shell in an atom decreases ANSWERS TO 2002 NURSING
outwards from the nucleus
e. The electrons in each have approximately
energy
shell equal
25. Concerning hard water
a. Spring water is the purest form of natural water
b. Rain water contains calcium sulphate
c. A hard water is one which will not readily form a
lather with
d. Sea water has the highest content of solid impurities
e. Soap is made from steam heating fat and caustic soda
solution
26. I saw you walking in that direction but
I did not know exactly
a. Where are you going?
b. Where you were going
c. Where were you going to?
d. To where were you going
e. Where you are going
27. The hill behind the town was so steep
a. That no man could find a foot holds on
b. That any man could find a toot hold on it
c. For a man to find a hold on it
d. That no man could find a hold
e. For any man could find foot
hold on.
28. Walking down the street
a. The boy was knocked down by
b. A car knocked the boy down
c. He knocked down the car
d. A car collided with the boy
e. A serious accident happened
29. The project which seemed very near
realization, because of lack of funds
a. Fell over b. Fell through c. Fell off
d. Fell in e. Fell down
30. The chairman of the company was very sympathetic
his staff
a. to b. at c. for d. from e. on
31. A machine has an efficiency of 60%. If the machine
is required to overcome a load of 30N with a force of
20N, calculate its mechanical advantage
a. 0.7 b. 0.9 c. 1.5 d. 2.5 e. 10.0.
32. A bus traveling at 15m-1, what is the distance
covered in 10s?
a. 350Cm b. 150Cm c. 70cm d. 250cm
e. 600Cm
33. Which of the following media allow(s) the
transmission of sound wave through them? I. Air II.
Liquid III. Solid
a. l only b. II only c. III only d. I & Il only
e. I, II & III
PAST-QUSTIONS 15. A True B. True C. False D. False E. False
1. A. False B True C. False D. False E. Obesity refer to a body mass index (BMI)
False The Rwanda genocide is a massacre greaten than 30. BMI is calculates as
!"#!! (!" !")
of the Tutsi by the Hut majority it lasted !"#$!! !"#"$% !
between April 1994 and July 994
2. A. False B. False C. False D. True E. False
Okodigbo succeeded Ewerem after the latter was
charged with falsification of his first name (Evan or
Evans) Qkadigbo was Io later impeached. however
both remained in the senate from 1999 to 2013.
3. A. True B. True C. True D. True E. True
4. A. False B. True C. False D. False E. False
5. A. False B. False C. False D. False E. False
The Nigeria senate has 109 members i.e 3 per state
plus one from FCT.
6. A. False B .False C. True D False E True
The other qualification were Nigeria, Cameroon
and Tunisia
7. A. False B. False C. True D. True E. False
Obasanjo was imprisoned by Gen. Sani Abacha and
was released after Abacha4s death (on 8 June, 1998).
He was initially sentenced to
death, later commuted to 30 years
imprisonment.
8. A. False B. True C. False D. False E. False James
jibpla Idowu Ige (SAN) was murdered in December
2001.
9. A, True B, True C True D True E
True Sharia law wass introduced in
Nigeria in 1999 by Governor Ahmad Sani
Yerima of Zamfara state.
10 A. False B. False C. True b False E. False
The Nursing and Midwifery council
of N9erJa was established by decree
N I79 now known as Nursing and
Midwifery (Registration etc) act.
Cap.
N143 Laws of the Federation of
Nigeria. 2004
11. A False B. False C, False D ,False E False
12, A. False B .False C .True D. False E.
False Mitochondria are for energy generation.
Lysosomes ar concerned with
destruction of old orgarielles and
certain compound.
13. A ,False B. True C False D .False E. True
At the end of the question, there is an
except. ‗This makes all true
statements, false and all false
statements true.
14. False B False, C, False D. True E. False
The Bile duct joins the pancreatic
duct‘, Which then open into the lumen
of the small intestine
16. A, False B. True C. False D .False E. False
The tail of a monkey also plays a role on balancing it n
trees.
17. A. True B. True C. False D. True E. False Rhesus
positive men who marry Rhesus negative women require
counseling.
18. A. False B. False C. True D. False E. False
19..A. False B. True C. False D. False E. True
Cellulose cell wells are part of plant cell
20. A. False B. False C. True D. False E. False
The hormones is called adrenaline (epinephrine)
21. A. True B. False C. False D. False E. True
Synonyms of deliberately intentionally, on purpose,
purpose, purposely, knowingly, consciously,
purposefully, wittingly, etc.
22. A. False B. False C. True D. False E. False
Synonyms of careless, inattentive, incaution, negligent,
remiss, causal, effortless, carefree.
23. A. True B. True C. False D. False E. False
Synonym of knowledgeable; well informed, informed,
learned, well read, educated, scholarly.
24. A. True B. False C. True D. False E. False
Electrons in a shell have different energies
25. A. False B. False C. True D. True E. True
Rain water is the purest form of natural water. Hardness
of water is caused by the presence of calcium and/or
magnesium ions in water.
26. A. False B. True C. False D. False E. False
27. A. False B. False C .False D. True E. False
28. A. True B. False C. False D. False E. False
29. A. False B. True C. False D. False E. False
30. A. True B. False C. False D. False E. False
31. A. False B. False C. True D. False E. False
!"#!!"#!$ !"#$ !" = 1.5
= =
!"#$%&$'( !""#$% !"
32. A. False B. True C. False D. False E. False
!"#$%&'(
Speed =
!"#$
Therefore, Distance = speed x time
Distance = 15 x 10 = 10m.
33. A. False B. True C. False D. False E. False

You might also like